Warum gilt die kalte Fusion als Schwindel?

Kalte Fusion wird in letzter Zeit viel erwähnt, weil ein neues Setup anscheinend funktioniert. Dies ist eine unbestätigte Behauptung.

Siehe zum Beispiel:

Während wir der wissenschaftlichen Gemeinschaft Zeit geben sollten, den Aufbau zu evaluieren und schließlich die Ergebnisse zu replizieren, besteht zweifellos eine gewisse Skepsis, dass die kalte Fusion überhaupt funktionieren würde, da die Behauptung ziemlich außergewöhnlich ist.

Nachdem Fleischmann und Pons in der Vergangenheit ihre Kaltfusionsergebnisse in gutem Glauben bekannt gegeben hatten, wurden sie durch nachfolgende Experimente als falsch erwiesen.

Was sind die experimentellen Realitäten, die dazu führen, dass Experimente mit kalten Fusionen im Stil von Fleischmann und Pons leicht falsch gemacht werden?

Würden die gleichen Risiken für diese neue Einrichtung gelten?

Ihr Journal of Nuclear Physics- Link gibt mir 403 zurück, und ich kann auf der Website keinen anderen finden. Die Website sieht so aus, als ob das Journal zum Zwecke der Berichterstattung über kalte Fusion existiert.
Abgesehen von technischen Problemen sollte dieses "Papier" von Rossi und Focardi alle möglichen verrückten Alarmglocken in Ihrem Kopf zum Klingen bringen: das dilettantische Schreiben, die fehlende Beschreibung des Versuchsaufbaus, der Mangel an Daten, die Betonung der Erfindung und der Patente, die Tatsache, dass das "Journal of Nuclear Physics" nichts anderes zu sein scheint als jemandes Blog ...
@dmckee,@nibot Bei mir ist es jetzt auch 403-ing. Ich weiß, dass das „Journal“ keine maßgebliche Veröffentlichung ist. Sie geben zu, dass sie es eingerichtet haben, weil sie nicht veröffentlicht werden konnten (sie gaben dem CF-Stigma die Schuld). Auf jeden Fall enthielt das Papier nicht den experimentellen Aufbau (sie sagen nur, dass sie im Grunde ein Kalorimeter verwendeten), aber der Aufbau ist in der Patentanmeldung gezeigt. Ja, das klingt für mich auch nach Blödsinn.
"funktioniert anscheinend" "vollkommen gutgläubig" Sie sind viel zu subjektiv. Der Grund, warum die meisten Physiker sehr skeptisch sind, liegt darin, dass es theoretisch keinen Sinn ergibt.
Ein eigenes Tagebuch ist nicht das Ende des Weges, aber es lädt zu einer genauen Prüfung ein. Zu wissen, dass man unter der Lupe ist und weniger als tadellose Arbeit abliefert, schafft kein Vertrauen.
@ Jeff: Theoretisch macht es keinen Sinn, und die experimentellen Beweise sind bestenfalls spärlich.
Warum die knappe Abstimmung? Ich habe das durch Meta laufen lassen und es scheint mir, dass die Frage objektive Antworten liefert?
@Sklivvz: Ich denke, es ist auch eine nette Frage, die zur Site gehört. Grüße
Die Tatsache, dass kalte Fusion ein Haufen dampfender Scheiße ist, ist keine Rechtfertigung dafür, die Diskussion darüber zu beenden, warum es ein Haufen dampfender Scheiße ist.
Denken Sie auch daran, dass Patente für originelle Ideen nicht funktionieren. Auch das US-Patentamt erteilte Tausende von Patenten für Perpetual Mobile Machines. Ein Patent bedeutet in diesem Zusammenhang nichts.

Antworten (14)

Die Behandlung von Pons/Fleischmann ist mit Abstand der schlimmste Skandal in der modernen Wissenschaft. Es ist wahrscheinlich schlimmer als Galileo. Ihre Behauptungen waren wahr, wurden sofort in mehreren Labors reproduziert (sporadisch fielen auch viele Labore aus), und gute Forschung wird bis heute fortgesetzt, ohne Finanzierung und ohne Theorie.

Experimentelle Geschichte

Ihr Anspruch ist nicht ganz neu. Es geht auf die 1920er Jahre zurück, als Paneth und Peters über die Heliumproduktion in der Pd-Schwerwasserelektrolyse berichteten. Die Behauptungen von Paneth und Peters wurden im Wesentlichen aus dem gleichen Grund wie die von Pons und Fleischmann zurückgewiesen, die Vorstellungskraft der Theoretiker war zu verkümmert, um sich einen Mechanismus auszudenken, der die Kluft zwischen chemischen Energien und nuklearen Energien überbrücken könnte. In den 1950er Jahren gab es einen sowjetischen Wissenschaftler, der auch behauptete, dass die Pd-Schwerwasserelektrolyse zu nuklearen Anomalien führt. Diese Behauptung wurde auch vom sowjetischen wissenschaftlichen Establishment zurückgewiesen, aber seine Karriere wurde etwas rehabilitiert, nachdem die Amerikaner den Effekt 1989 reproduziert hatten.

Viele Leute (z. B. Doktoranden), die jahrzehntelang mit dem Pd/Deuterium-System gearbeitet haben, bemerkten Anomalien im System, und es war Folklore in der Chemie-Community, dass deuteriertes Palladium wirkt, während hydriertes Pd dies nicht tut. Pons und Fleischmann beschlossen, sich ernsthaft mit den Anomalien zu befassen, und führten viele Jahre lang äußerst sorgfältige Kalorimetrie an dem System durch, bis sie sicher waren, dass sie einen reproduzierbaren Effekt hatten, für den die Chemie sicher ausgeschlossen werden konnte. Dann hielten sie ihre Pressekonferenz ab und Chaos.

Ein Teil des Problems besteht darin, dass die Leute, nachdem sie die Fusion behauptet hatten, darauf bestanden, dass die Fusion Neutronen emittieren sollte, genau wie die heiße Fusion. Das ist unmöglich, denn angesichts der freigesetzten Energie hätte die Zahl der Neutronen Pons und Fleischmann gekocht. Dann verlangten Nuklearphysiker, dass sie nukleare Effekte messen sollten, und sie versuchten es auch, aber ihre nuklearen Messungen waren voller Fehler, und es ist möglich, dass sie eine Verschwörung, die sie auf einer Konferenz gezeigt hatten, verfälschten (obwohl angesichts von Fleischmanns tadelloser wissenschaftlicher Integrität I finde es plausibler, dass sie einen ehrlichen Fehler gemacht haben). Es ist wichtig anzumerken, dass ihre veröffentlichte Arbeit nur Kalorimetriedaten enthält und keine nuklearen Daten, bei denen sie sich nicht sicher waren.

Einige Leute spekulierten, dass der Effekt durch die Chemie oder durch unzureichendes Rühren oder durch das Speichern elektrischer Energie für eine spätere Freisetzung erklärt werden kann. Diese Behauptungen sind alle idiotisch. Der Effekt ist nicht gering, der einzige Grund, warum Instrumente zum Nachweis erforderlich sind, liegt darin, dass Pons und Fleischmann absichtlich einen winzigen Pd-Draht als Kathode verwendeten. Als sie eine größere Pd-Platte verwendeten, schmolz das Ding den Tisch und blies ein Loch in den Betonboden darunter. Es gibt einfach keine chemische Energiequelle und keine Batterie, die Energie chemisch mit mehr als etwa 1 eV pro Atom speichern kann. Andere Leute bemerkten auch ähnliche außer Kontrolle geratene Explosionen.

Abgesehen von der Explosion gibt es eine mögliche Wärme durch Rekombination, die von Kritikern oft betont wurde. Die Elektroden trennen H2 von O2, und wenn sich die beiden vermischen und der Wasserstoff verbrennt, sehen Sie überschüssige Wärme. Um dies zu kontrollieren, verwendeten Gruppen Infrarotkameras, um die Wärmequelle an der Kathode zu lokalisieren und nicht das Wasser, wo sich Gasblasen vermischen können. Sie trennten auch Anode und Kathode. Aber am definitivsten demonstrierten Pons und Fleischman 1994 Hitze nach dem Tod im Jahr 1994, wo sie die Temperatur in der anomal Heizzelle hochfahren und dann den Strom vollständig abschalten. Die Zelle produziert stundenlang ohne Strom , ohne Sauerstoff, ohne Wasserstoff und um ein Vielfaches mehr Wärme, als Sie mit chemischen Mitteln in der Kathode speichern können.

Der Effekt reagiert sehr empfindlich auf die Metallurgie des Palladiums, und Pons und Fleischmann konnten den kniffligen Effekt auf Nachfrage nicht reproduzieren, als ihnen das gute Palladium ausging. Das Experiment dauert manchmal Wochen, und viele Leute hatten einfach keine Geduld. Dennoch wurde der Effekt an einigen wenigen Stellen sofort reproduziert. Das MIT betrieb eine berüchtigte Reproduktion, die eine übermäßige Wärmeentwicklung bemerkte, und wollte eine Reproduktion herausbringen. Dann wurde ihnen klar, dass dieser Effekt als falsch bezeichnet werden würde, und sie schnitten ihr Diagramm ab, um keine übermäßige Wärme zu zeigen. Einer der Doktoranden, der an diesem Experiment beteiligt war, Eugene Mallove, war so empört, dass er seine Stelle kündigte und Förderer der Kalten Fusion wurde.

Mehrere Gruppen veröffentlichten Reproduktionen. Diese Gruppen wurden auf höchst unwissenschaftliche Weise angegriffen. Mehrere Gruppen, Bocris in Texas, aber auch angesehene Forscher am Bhabha Institute und Los Alamos, berichteten von einer geringen Tritiumproduktion im System. Da Tritium radioaktiv ist, hat es eine klare Signatur, es kann mit nichts anderem verwechselt werden. Da es sehr teuer ist und normalerweise in Kernreaktoren erzeugt wird, könnte ein solches Signal nur gesehen werden, wenn es absichtlich gefälscht wurde, indem das schwere Wasser mit Tritium versetzt wurde. Bocris wurde beschuldigt, genau das getan zu haben – seine Zellen mit Tritium versetzt zu haben, so überzeugt waren die Leugner, dass er Betrug begangen hatte. Trotz des intensiven Drucks zog er seine Behauptung nie zurück. Ein anderer Kollege in Texas, der Tritium behauptete, Wolf, zog die Behauptung zurück, als er sah, was mit Bocris geschah. und sprach nie wieder für die kalte Fusion. Bocris wurde wegen wissenschaftlichen Fehlverhaltens untersucht und entlastet. Es wurde nie ein plausibler Weg gefunden, wie er Tritium (außer kalter Fusion) erhalten könnte. Die Tritium-Beobachtungen erfordern würdenalle Forscher, die beobachteten, dass Tritium in vorsätzlichen Betrug verwickelt war. Es ist unmöglich, Tritium falsch zu identifizieren.

Zwei äußerst angesehene Theoretiker, Julian Schwinger (emeritus UCLA) und Peter Hagelstein vom MIT, waren überzeugt, dass der Effekt real war. Schwinger wurde nicht erlaubt, auf diesem Gebiet zu publizieren, und Hagelstein, der angestellt war, wurden alle seine Mittel gekürzt und er wurde in einen Schrank verlegt.

In den frühen 90er Jahren quantifizierte McKubre ohne offizielle Finanzierung die Heliumproduktion, um sie mit der überschüssigen Wärme zu korrelieren. Im System treten geringe Mengen an gewöhnlicher Fusion auf. Die SPAWAR-Gruppe der US-Marine reproduzierte den Effekt in Koabscheidungsexperimenten, bei denen sie Pd in ​​Gegenwart von schwerem Wasser auf eine Oberfläche aufbringen. Ihre Experimente detektieren hauptsächlich Nuklearprodukte, weil die plattierte Oberfläche so klein ist, aber die Effekte sind zu 100% reproduzierbar. In jüngerer Zeit legte die Marine Beweise für sporadische hochenergetische Neutronen vor, die aus dem Co-Depositionssystem stammen.

In Japan bemerkte Mizuno, dass neue Elemente im Pd-System mit abnormalen Isotopenverhältnissen und Ordnungszahlen in der Nähe von Pd produziert wurden (dies wurde auch von Wolf entdeckt, wie von Eugene Mallove berichtet, aber Wolf wollte nach dem Tritium-Fiasko nichts veröffentlichen). Anormale Isotopenverhältnisse können kein Betrug sein, weil solche Materialien so schwierig herzustellen sind. In Japan reproduzierte Arata den Effekt, indem es eine Gasbeladung von Deuterium in Pd verwendete, das keine Wärmequelle hat, sodass es keinen Kalorimetriefehler gibt, dem die Schuld gegeben werden könnte. Dies war eine idiotensichere Version von Pons' und Fleischmanns "Hitze nach dem Tod"-Experimenten und schließt auch Kalorimetrie-/Rekombinationsfehler vollständig aus. Der Effekt wurde viele hundert Male reproduziert, in weit entfernten Labors ohne gemeinsame Interessen, und jeder sollte inzwischen sicher sein, dass er echt ist. Ich schäme mich,

Focardi/Rossi-Behauptungen sind zweifelhafter. Ihre Wirkung zeigt sich in Nickel/Wasserstoff, das ebenfalls einige gemeldete Energieanomalien aufweist, jedoch nicht mit dem gleichen Maß an Vertrauen. In Bezug auf den theoretischen Wahnsinn verhält sich die Nickel-Wasserstoff-Fusion zur Palladium-Deuterium-Fusion wie die Palladium-Deuterium-Fusion zur Standard-Heißfusion. Die Cold-Fusion-Community nimmt eine abwartende Haltung ein, aber ich denke, Konsens besteht darin, dass das Gerät wahrscheinlich nicht funktionieren wird. In seinen Demonstrationen hat Rossi die Wärmeerzeugung mit Dampf gemessen, nicht mit Wasser, und indem man den Wassergehalt des Dampfes unterschätzt, kann man die Energieabgabe durch die latente Verdampfungsenergie aufblähen, die enorm ist. Für mich ist es am verdächtigsten, dass seine behaupteten Transmutationsprodukte analysiert wurden und natürliche Isotopenverhältnisse aufweisen. Es ist möglich, dass seine Maschine funktioniert, und es ist möglich, dass es überhaupt keine überschüssige Energie produziert, werden wir bald genug wissen. Unmöglich ist, dass es bei Pd/Deuterium keine nuklearen Effekte gibt.

Hier ist ein Update bezüglich der E-Katze, die erwartungsgemäß ein ausgeklügelter Betrug ist: Ist die E-Katze von Andrea Rossi et al. wirklich? .

Theoretische Arbeit

Eine Hauptschwierigkeit für die Akzeptanz des Effekts besteht darin, dass die theoretische Arbeit auf diesem Gebiet nicht fundiert ist. Es gibt mehrere Theorien, von denen jede mehr oder weniger absurd ist. Die zentralen Schwierigkeiten bestehen darin, die Coulomb-Barriere irgendwie zu überwinden und Energie ohne Nebenprodukte von Kernreaktionen zu erzeugen:

  • Hydrinos/kleiner Wasserstoff: Diese Theorie besagt, dass das Elektron in Wasserstoff eine nähere Umlaufbahn als der Grundzustand finden kann und einige Zeit in der Nähe des Kerns verbringt. Dies erfordert, dass die Quantenmechanik falsch ist oder dass es eine neue Elektron/Proton-Kraft gibt, die übersehen wurde und irgendwie die Grundzustandsenergie nicht verändert, aber in der Lage ist, das Elektron von Zeit zu Zeit in das Proton zu saugen.
  • Bose-Einstein kondensierte Deuteronen/Alphas: Diese Idee ist, dass der Wirkungsquerschnitt für die Fusion durch Effekte identischer Teilchen verstärkt wird, da Deuteronen und Alphas beide Bosonen sind. Theoretisch können Sie Reaktionen verstärken, indem Sie eine kohärente Quelle von Bosonen haben, die alle dieselbe Reaktion auf eine kohärente Überlagerung durchlaufen. Diese Theorie versagt sowohl, weil die Temperatur für eine Kohärenz zwischen Deuteronen zu hoch ist, als auch, weil, wenn sie in bestimmten Kaltfusionspapieren implementiert wird, die Deuteronen als nicht wechselwirkende Teilchen in einem Produktzustand behandelt werden, so dass die Amplitude gleich ist Punkt ist groß. Aber das ignoriert die ganze Schwierigkeit, weil die elektrostatische Abstoßung dazu führt, dass die Wellenfunktion verschränkt wird, mit geringer Wahrscheinlichkeit, dass zwei Deuteronen an denselben Punkt gelangen.
  • Gitterverstärkungsmechanismen: Dies war der Fokus von Schwinger und Hagelstein, von denen keiner behauptete, das Problem gelöst zu haben. Das Problem bei solchen Theorien ist nur, dass die Effekte über Tausende von Atomen kollektiv sein müssen, um die Umrechnung von eV-Energien in KeV-Energien zu erklären, und es ist thermodynamisch schwer vorstellbar, wie man eine solche entropische Energie als einzelne an einen so entropisch ungünstigen Ort bringen kann Partikel.
  • Erzeugung schwacher Neutronen: Die Widom-Larson-Theorie besagt, dass es für ein Proton und ein Elektron möglich ist, einen inversen Beta-Zerfall auf der Oberfläche eines Metalls durchzuführen, wo große lokale elektrische Felder vorhanden sind. Dies ist wegen des MeV-Unterschieds in der Protonen- und Neutronenmasse absurd. Es sind Millionen Volt erforderlich, um ein Elektron auf genügend Energie zu beschleunigen, um einen inversen Beta-Zerfall durchführen zu können, und solche Energien sind auf der Oberfläche eines Metalls nicht verfügbar. Darüber hinaus sagt diese Theorie überwiegend Transmutationen von plus/minus einer Masseneinheit voraus, was nicht beobachtet wird, und erklärt nicht, wie ein Deuteron ein Elektron absorbieren kann.

Die folgenden Listen sind falsche Theorien, von denen ich spekulierte, dass sie funktionieren würden, andere Leute kommen auch hin und wieder auf diese:

  • sporadischer atmosphärischer Myoneneinfang: Die Idee dahinter ist, dass Myonen vom Metall eingefangen werden und zur Fusion führen. Das funktioniert nicht, nur weil nicht genügend Myonen vorhanden sind, die Deuteronen im Gitter voneinander getrennt sind und das Myon, wenn es von einem Pd-Kern eingefangen wird, verschwendet wird.
  • Tunneln mit seltsamer Vielteilchenverstärkung: Die Idee ist, dass die Tunnelamplitude immer geschätzt und nicht berechnet wird, und dies ein unmöglich zu lösendes System mit vielen Elektronen / vielen Nukleonen ist, sodass die Tunnelamplitude möglicherweise nur um viele Größenordnungen davon abweicht Größe. Dies funktioniert nicht, weil es eine Möglichkeit gibt, den Tunnelamplituden eine untere Grenze zu geben, die jede nennenswerte Fusionsreaktion durch Tunneln ausschließt. Dazu nutzt man die Tatsache aus, dass das Tunneln eine Grundzustandseigenschaft ist und die Deuteronen, die man sich zum Tunneln vorstellt, Bosonen sind und ihr imaginärer Zeitgrundzustand keine Knoten hat. Die Elektronen haben Knoten, da sie Fermionen sind, und bei hohen Energien, aber wenn die Elektronenzustände alle vollständig besetzt sind, könnten sie genauso gut ein Vakuum sein, mit Struktur nur in der Nähe der Fermi-Fläche (dies folgt aus der Teilchen-Loch-symmetrischen ungefähren Beschreibung der Fermi-Flüssigkeit). Es gab strenge Obergrenzen für die Tunnelwahrscheinlichkeit von Deuteronen in einem Metall, das angeblich bewies, dass eine kalte Fusion unmöglich ist.

Die vorherigen Fehler lassen vermuten, dass der Effekt aus dem Gleichgewicht geraten ist und hoch angeregte Atome betrifft.

Meine persönliche Theorie

Um die Lücke zwischen der Skala der Chemie bei eV und der Kerne bei MeV zu überbrücken, sollte man beachten, dass es K-Schalen-Elektronen gibt, die bei KeV-Energien sehr nahe am Kern kreisen. Das Elektron der K-Schale von Pd hat eine Ionisationsenergie von 20 keV, und wenn Sie ein Loch in der K-Schale in einem Pd-Atom haben, speichert es eine Energiemenge nicht-entropisch in einer Menge, die ausreicht, um zur Deuteronfusion zu führen. Obwohl diese Energie groß ist, ist sie nicht groß genug, um ein Palladiumatom aus seiner Gitterposition zu schlagen, sodass es seine Energie nicht durch lokales Aufbrechen des Gitters abgeben kann. Der Grund dafür ist, dass der Pd-Kern mehr als die 20-KeV-Ionisierungsenergie benötigt, um ohne seinen Kern aus der Position geschlagen zu werden, und Sie die Lochenergie nicht in einem Schritt verschwörerisch auf den gesamten Kern übertragen können, es ist ein Phasenraum unmöglich.

Solche K-Schalen-Löcher zerfallen normalerweise durch Röntgenstrahlen, aber dies ist ein elektromagnetischer Prozess, der durch Potenzen von v/c unterdrückt wird, wenn das Elektron nichtrelativistisch ist, wie es sogar in der K-Schale der Fall ist. Dies ist ein wohlbekannter Effekt – es ist der gleiche Grund dafür, dass atomare Spektrallinien schmal sind. Das Emittieren eines Photons dauert viele Umlaufbahnen, da der Maßstab zwischen der Wellenlänge des Photons und der Größe der Umlaufbahn nicht übereinstimmt. Dies liegt letztendlich daran, dass die Umlaufbahn nichtrelativistisch ist. Da die Emission so lange dauert, sind die Spektrallinien scharf definiert und schmal, und die Emission wird von den Matrixelementen des Dipolmoments des atomaren Zustands zwischen stationären Zuständen dominiert.

Andere beobachtete Möglichkeiten für K-Schalen, ihre Energie zu verlieren, bestehen darin, ein Elektron der äußeren Schale aus einem benachbarten Atom herauszuschlagen. Dieser Prozess ist elektrostatisch und nichtrelativistisch, wird also nicht durch 1/c-Faktoren unterdrückt. Sie wird nur durch die geringe Ladung des Elektrons und den Abstand zwischen Elektronen benachbarter Atome unterdrückt. In diesem Kanal gibt es einen signifikanten Anteil an Zerfällen in K-Löchern in Pd.

In einem Metall mit Protonen oder Deuteronen sollte ein K-Schalenloch seine Energie durch elektrostatische Kräfte auch in ein Proton oder Deutronen kicken können. Das Matrixelement ist genau dasselbe wie beim Treten eines Elektrons, aber die Zustandsdichte ist aufgrund der höheren Masse 30-50 mal größer (je nachdem, ob es sich um ein Proton oder ein Deuteron handelt). Das Proton wird im Gegensatz zu einem Pd-Kern bei einem solchen Transfer seinen Gitterplatz verlassen. Wenn man bedenkt, dass der Querschnitt für ein K-Schalen-Loch zum Treten eines Elektrons nicht klein ist, kann ich mit Sicherheit zu dem Schluss kommen, dass der Protonen-Kick-Prozess der dominierende Zerfallsmechanismus für K-Löcher ist.

Diese Deuteronen haben genau die gleiche Energie wie das Loch der K-Schale, was bedeutet, dass ihr klassischer Wendepunkt bei Annäherung an einen Pd-Kern elektrostatisch genau so weit vom Kern entfernt ist, wie die K-Schale breit ist, etwa 100 Fermis. Diese Löcher können dann ein weiteres Elektron kohärent anregen und viele Schritte im Gitter zurücklegen, bevor sie durch Röntgenstrahlen in den Grundzustand zerfallen. Diese Loch-Deuteron-Zustände bilden Bänder mit einer Breite von mehreren KeV bei Energien um 20 KeV, und diese Bänder sind voll von klassischen Wendepunkten bei 100 Fermis von einem Pd-Kern.

Nehmen wir nun an, dass zwei dieser beschleunigten Deuteronen zufällig in die Nähe desselben Pd-Kerns kommen. Dies kann am Wendepunkt leicht zu einem Fusionsereignis führen, die Deuteronen haben immerhin ungefähr 20 KeV, und die Fusionsraten bei 20 KeV in Strahlen sind nicht so klein, geschweige denn in Fällen, in denen die Wellenfunktion in der Nähe eines Kerns mit einer klassischen Drehung konzentriert ist Punkt (wo die Wellenfunktion verstärkt wird).

Diese Fusion erfolgt nicht unbedingt auf die übliche Weise der heißen Fusion, da sie sehr nahe an einem Pd-Kern liegt. Nehmen wir an, dass die Fusion die überschüssige Energie/Impuls elektrostatisch auf ein nahe gelegenes geladenes Teilchen überträgt, wobei der offensichtliche Kandidat einer der Protonen-Pd-Kerne ist. Dann bewegen sich das Alpha-Teilchen und was auch immer es seine Energie übertragen hat, zusammen mit einer Energie von 24 MeV, und sie gehen durch das Metall und ionisieren Pd-Atome. Energetisch können sie bis zu 1000 K-Shell-Löcher bohren, alle innerhalb eines Millimeters, da die Eindringtiefe so gering ist. Die wahre Zahl liegt eher bei hundert oder einigen hundert, da alle Niveaus während des Bethe-Prozesses der Ionisation geladener Teilchen angeregt werden. Diese Löcher werden dann mit Deuteronen gebändert, sodass sie neue Deuteronen beschleunigen, was leicht zu einer Kettenreaktion führen kann.

Es gibt zwei Probleme mit dieser Idee:

  1. Der Wirkungsquerschnitt für die Fusion bei 20 KeV ist nicht so groß und führt nicht zu einer Kettenreaktion durch die üblichen Hot-Fusion-Kanäle. Der Multiplikationsfaktor liegt bei etwa 0,001 von der Strahlfusion auf deuteriertem Pd, das bei 20 KeV eine Erfolgsrate von 1 zu 100.000 und nicht 1 zu 100 hat.
  2. Die tatsächlich beobachtete Reaktion erzeugt fast immer ein Alphateilchen ohne ein emittiertes Neutron oder Proton. Dies ist ein 1-in-1-Million-Ereignis in der heißen Fusion.

Ich denke, dass beide Probleme damit zusammenhängen, dass die Reaktion in einem dichten Metall stattfindet. Das erste Problem ist nicht vorhanden, wenn zwei Deuteronen gebändert sind und sich beide in der Nähe eines Kerns umdrehen, ist das Ergebnis wie eine gerichtete Kollision von zwei 20-KeV-Strahlen mit einer sehr guten Fokussiervorrichtung (dem Kern), um die streuende Wellenfunktion zu konzentrieren.

Die Fusion von Deuteronen geschieht immer durch instabile Zwischenzustände, und der Wirkungsquerschnitt zum Alpha-Teilchen ist aufgrund des gleichen nicht-relativistischen Problems nur klein. Um ein Alpha zu erhalten, müssen Sie ein Gammastrahlen-Photon emittieren, und die Emissionen von Photonen werden durch 1/c-Faktoren unterdrückt. Wenn sich ein Kern in der Nähe befindet, kann er elektrostatisch geschleudert werden, und dieser Vorgang ist einfacher als das Herausschleudern eines Photons, weil er nichtrelativistisch ist (dasselbe gilt für ein Elektron, aber mit viel kleinerem Querschnitt aufgrund der geringeren Ladung, und dort ist kein Grund, eine Konzentration der Wellenfunktion um die Elektronendichte herum zu vermuten, wie dies für einen Kern der Fall ist).

Die Zeitskala für das Treten eines Kerns ist die Lebensdauer der Zwei-Deuteronen-Resonanz, die nicht sehr lang ist, in Bezug auf die Entfernung beträgt sie etwa 100 Fermis, das ist ungefähr so ​​​​groß wie die innere Hülle. Wenn die Deuteronen zufällig umherschwirren, ist diese Koinzidenz nicht signifikant, aber wenn die Deuteron-Loch-Anregungen gebändert sind, ist es plausibel, dass fast alle energetischen Deuteron-Deuteron-Kollisionen sehr nahe an einem Kern stattfinden, wie oben erklärt.

Es gibt Erhaltungsgesetze, die gebrochen werden, wenn ein Kern in der Nähe ist. Der Kern bricht die Parität, also könnte er einen Fusionskanal öffnen, indem er Deuteronenpaaren erlaubt, von einem ungeraden Paritätszustand zu einem Alpha zu zerfallen. Solch ein Übergang würde niemals in einer Dilute-Beam-Fusion beobachtet werden, da diese Fusionen weit entfernt von allem anderen stattfinden. Diese Hypothese wird durch die Alpha-Teilchen-Spektroskopie nicht ausgeschlossen (es gibt viele relevante Niveaus unterschiedlicher Paritäten), aber sie wird auch nicht vorhergesagt.

Aber da irgendetwas die experimentellen Daten erklären muss und diese Idee die einzige Geschichte ist, die nicht ganz weit hergeholt ist, glaube ich, dass genau das vor sich geht.

Diese Theorie sagt Folgendes voraus

  • Bei der kalten Fusion sollte das Material reichlich Röntgenstrahlen im KeV-Bereich emittieren (von den Löchern der K-Schale, die sowieso elektromagnetisch zerfallen).
  • das Material sollte KeV-Deuteronen in einer mm Haut um es herum emittieren.
  • das Material sollte Alphas im MeV-Bereich und Pd-Kernfragmente, Protonen, Deteronen, Tritium und Pd-Fusionsprodukte emittieren, je nach Eignung für ~10MeV-Elektronenstreuung. Die Alphas sollten bis zu 20 MeV hochgehen, was die maximale Energie ist, wenn der gesamte Kern gestreut wird. Die Pd-Fragmente sollten MeV-Energie haben.
  • Es sollte eine kleine Menge heißer Fusion stattfinden, mit den dazugehörigen schnellen Neutronen und Tritium, nur von gelegentlichen zufälligen heißen Fusionskollisionen von 20-KeV-Deuteronen weit entfernt von einem Kern. Wenn die Bänder inkohärent werden, kann es zu einem Ausbruch von Neutronen kommen, da die inkohärenten schnellen Deuteronen zufällig verschmelzen.

Dies waren nur insofern „Vorhersagen“, als ich sie nicht kannte, als ich die Theorie aufstellte. Ich habe auf lenr-canr.org herausgefunden, dass 1,3,4 von Mosier-Boss unter Verwendung von CR-39-Plastikdetektoren und Röntgenfilmen zusammen mit anderen beobachtet werden, obwohl es schwierig ist, ein KeV-Deuteron zu sehen.

Die Theorie sagt auch Folgendes voraus

  • Protonenbasierte kalte Fusion funktioniert nicht (obwohl es möglicherweise eine Möglichkeit gibt, KeV-Energien in einem Nickel-Wasserstoff-System für lange Zeit in K-Schalenbändern zu speichern und sie in Schüben freizusetzen, obwohl dies mir unwahrscheinlich erscheint). Dies erfordert, dass alle Überhitzungsberichte der Ni-H-Kaltfusion auf chemische Rekombination zurückzuführen sind und keines davon Nuklearprodukte aufweisen sollte. Dies steht nicht im Widerspruch zu irgendwelchen Daten, die ich gesehen habe.
  • Transmutationsprodukte bei der kalten Fusion sind auf Pd-Fragmentierung während der Fusion und schnelle Alpha-Absorption/-Streuung oder schnelle Pd-Fragment-Absorption/Streuung zurückzuführen.

Die einfachsten Fragmentierungs- und Alpha-Absorptionsvorhersagen bedeuten, dass Sie den Pd-Übergang zu Au (+1, von Alpha-Absorption und Protonenausstoß oder Alpha-Absorption, Gamma-Emission und Beta-Zerfall) und +2 Cd (von Alpha-Absorption und Gamma-Emission) beobachten sollten, aber nicht höher und Übergänge nach unten sind auf Fragmentierung zurückzuführen, daher sollten Sie Pd-Spaltprodukte, Rh (von ausgestoßenen Protonen) und Ru (ausgestoßene Alphas) ​​sehen. Diese stimmen genau mit Wolfs Transmutationsdaten überein, die aus dem Gammaspektrum der Radioisotope stammten, die nach einem erfolgreichen Lauf in der Kathode vorhanden waren, wie von Eugene Mallove durchgesickert.

Diese Vorhersagen sind jedoch nicht mit allen auf lenr-canr.org präsentierten experimentellen Daten zu Transmutationen kompatibel. Ich glaube, dass die Transmutationsdaten insofern falsch sind, als sie dieser Theorie widersprechen.

Transmutationen

Die durch Pd forcierten Transmutationsprodukte in Deuterium zeigen Peaks bei Masse +8, +12. Bombardierung durch Alphas kann dies nicht bewirken, da es absolut unmöglich ist, dass dasselbe Atom zweimal von zwei verschiedenen Alphas getroffen wird.

Dies erfordert, dass ein fusionsfragmentiertes Pd ein Be8 als sein Fusionsfragmentierungsprodukt ausstößt und dass dieses Be8 dann während des Transits von einem anderen Kern absorbiert wird, wodurch 8 Masseneinheiten an einen anderen Kern durch Absorption abgegeben werden. Wenn dem so ist, gibt es Summenregeln für die Transmutationselemente: Die Menge des erzeugten leichten Elements X ist gleich der Differenz von (Pd+X) und (Pd-X), wobei Pd+X bedeutet, alle Protonen und Neutronen zu addieren in X zu Pd, und Pd-X bedeutet, alle Neutronen und Protonen in X von Pd abzuziehen. Diese Summenregel ist ein strenger Test der Theorie.

Wenn Sie außerdem davon ausgehen, dass die Absorptionswahrscheinlichkeit ungefähr geometrisch ist, muss sie genau dann abschneiden, wenn die Coulomb-Barriere 10 MeV überschreitet, was Transmutationen von bedeutet (dies ist vernünftig für die Absorption eines kleinen Kernprojektils durch einen großen Kern), können Sie daraus schließen Sie erkennt Peaks bei Pd+X, die systematisch um die gleichen Faktoren wie Pd-X verschoben sind.

Aber man würde erwarten, dass die schweren Transmutationsprodukte als Unterdrückung der Coulomb-Barriere abfallen. Dies ist angesichts der Beobachtungen von Iwamura nur qualitativ plausibel. Sie können die Massenspezifikationsergebnisse von Iwamura auf lenr-canr.org einsehen

Diese Theorie ist mehr oder weniger eine Massentheorie, daher ist es schwer zu verstehen, warum die Oberfläche wichtiger ist. Es wird vorhergesagt, dass Banden bei 20 KeV in deuterierten Metallen auftreten, und dies würde einen ganzen Zoo nützlicher Effekte ergeben, unabhängig vom nuklearen Zeug. Die Bandbreite verschiebt das Röntgenspektrum des deuterierten Metalls von 20 KeV in einen breiten Bereich, was auch eine leicht zu überprüfende Vorhersage der Theorie ist – die K-Schalen-Resonanzfrequenzen werden durch Deutration verändert.

Ich erzähle diese theoretische Geschichte, weil ich denke, dass sie plausibel ist und mit den Daten ohne neue grundlegende Physik übereinstimmt, sodass die Leute die kalte Fusion nicht abtun sollten. Da keine andere Erklärung auch nur annähernd funktioniert, vermute ich, dass diese Erklärung richtig ist.

Probleme mit der Theorie

Das Hauptproblem mit der Theorie ist die Unvollständigkeit, es ist eine Skizze. Aber die Hauptpunkte der experimentellen Daten stehen nicht mehr im Widerspruch zur Theorie. Dies ist eine größere Änderung des ursprünglichen Beitrags, den ich vor einiger Zeit veröffentlicht habe. Zum Zeitpunkt des ursprünglichen Postings kam mir nicht in den Sinn, dass die elektrostatische Übertragung von 10 MeV Energie auf einen Kern zu einer Kernfragmentierung führen kann, und ohne dies ist die Theorie nicht mit den Transmutationsdaten vereinbar.

Hm, das klingt eigentlich ganz vernünftig. Von nun an werde ich etwas offener gegenüber der Kalten Fusion sein. Danke für die informative Antwort.
Sie scheinen ein bisschen voreingenommen für kalte Fusion zu sein, genauso wie ich dagegen voreingenommen bin. Ich habe für viele der Elektrochemiker gearbeitet, die die Arbeit von Pons/Fleischmann nicht reproduzieren konnten. Das bedeutet nicht, dass P/F falsch war, aber ich wäre sehr überrascht, wenn diese exzellenten Wissenschaftler, mit denen ich zusammengearbeitet habe, etwas übersehen hätten. Vielleicht haben Sie Geld/Zeit in die kalte Fusion investiert, wenn Sie es wirklich glauben, aber ich sichere meine Wetten ab, indem ich woanders investiere. Es spielt keine Rolle, wie gut Ihre Idee klingt, aber es ist wichtig, was tatsächlich funktioniert. Bisher sehe ich keinen Gewinner auf Ihrer Seite, aber möge die beste Idee gewinnen!
"Ich denke, dass beide Probleme damit zusammenhängen, dass die Reaktion in einem dichten Metall stattfindet." Ich denke, beide Probleme hängen damit zusammen, dass die Reaktion überhaupt nicht stattfindet.
@Chris: Ich bin nur für kalte Fusion voreingenommen, weil ich mich schuldig fühle, weil ich es nicht früher geglaubt habe. Ich habe kein persönliches Interesse, außer der Tatsache, dass ich glaube, dass die Erklärung, die ich oben gegeben habe, die richtige ist.
@Ben: Es ist wichtig, dass Sie sich die experimentellen Arbeiten ansehen, bevor Sie sich entscheiden, und sie mit einem kritischen Auge überprüfen (lenr-canr.org).
Wenn es ein echter Effekt ist, warum investieren Sie dann nicht ein paar tausend Dollar, damit Sie mit dem Effekt Wasser kochen, eine Turbine antreiben und Millionär werden können? Warum tut das niemand? Wenn Sie die kalte Fusion gefunden haben, <i>bauen Sie den verdammten Reaktor</i> oder zeigen Sie zumindest, dass Sie etwas Lithium oder Helium oder so etwas produziert haben. Es gäbe null Zweifler, wenn jemand einfach <b>den verdammten Reaktor bauen</b> würde.
Der Effekt ist schwer reproduzierbar. Pons und Fleischmann waren sich sicher, dass sie einen Reaktor bauen könnten, aber sie hatten Schwierigkeiten genug, den Effekt einmal pro Woche zu erreichen. Wenn ich Geld hätte, würde ich gerne ein Pd/d-System einrichten und versuchen, es mit Röntgenstrahlen zu betreiben, die auf die Übergänge zwischen den Innenschalen abgestimmt sind, um zu sehen, ob die Innenschalen etwas Seltsames tun. Persönlich würde ich mich freuen, genügend experimentelle Daten zu erhalten, um eine effektive Feldtheorie der Deuteron-Resonanzen in einem starken elektrischen Feld zu erstellen. Beides wäre eine interessante Physik für sich.
@RonMaimon: Sie mögen Widom-Larsen nicht, aber Sie mögen auch die Implikationen Ihrer eigenen bevorzugten Erklärung nicht, teilweise wegen der großen Bandbreite an Isotopen, die die Leute sehen. Ich verstehe, dass es eine Menge Forschung gibt, die sich mit einem besseren Verständnis der Metallizität von Sternen befasst hat, die sich aus der r-Prozess-Nukleosynthese sowie den Ergebnissen der Neutronenaktivierungsanalyse ergibt. Welches Potenzial sehen Sie in der Anwendung der Forschung auf diesen Gebieten auf die Spektroskopie der LENR-Experimente, um einige der Freiheitsgrade zu eliminieren, die bestehende Erklärungen beeinträchtigen?
@Eric: Der große Isotopenbereich ist äußerst rätselhaft, aber ich kann es nicht definitiv ausschließen. Es könnte denkbar sein, dass ein 20-MeV-Alpha/Pd-Bombardement die Kerne zerfetzt. Sie können diese Idee direkt testen, indem Sie Bombardierungsexperimente durchführen. Widom Larson ist ein No-Go – man kann keine Neutronen erzeugen, es ist einfach energetisch unmöglich. Ich finde alle vorhandenen Erklärungen ähnlich lächerlich, weshalb ich das Gefühl hatte, eine vernünftige Alternative anbieten zu müssen. Ich kenne den r-Prozess nicht, aber aus dem Wiki scheint er eine neutronenreiche Umgebung zu erfordern. Wie würde es übersetzt werden? Alle Ideen sind willkommen.
Ich dachte daran anzunehmen, dass die gemeldeten Spektren innerhalb eines vernünftigen Fehlers genau sind, und dann von dort aus rückwärts zu arbeiten, um potenzielle Zerfallsketten zu eliminieren. Mögliche Zwischenschritte würden dies zu einer schwierigen, aber aus der naiven Perspektive eines Neulings nicht zu einer unmöglichen Herausforderung machen. Wenn man Glück hätte, könnte man möglicherweise eine große Anzahl von Ketten ausschließen und eine Teilmenge übrig lassen, die verwendet werden könnte, um die Dinge ein wenig genauer festzulegen. Man müsste nicht zu viele Annahmen über den anfänglichen Prozess machen – Alpha-Bombardement usw. Der Fokus würde hauptsächlich auf dem Beta-Zerfall liegen.
@ Eric: Ich stimme zu, dass Sie bestimmte Ketten eliminieren können. Ich bin mir nicht sicher, ob es eine vollständige Darstellung des Spektrums der Pd / d-Reaktion gibt, die quantitativ ist. Wenn Sie sie finden, würden Sie einen Link posten? Die, die ich gefunden habe, war eine qualitativere Spektrometrie zur erzwungenen Deuteriumdiffusion durch Pd, wenn ich mich richtig erinnere.
@Ron: Hier ist ein Link zu einem Artikel mit der Art von Daten, von denen ich dachte, dass sie interessant zu analysieren wären: bit.ly/rTA47W . Sie kombinieren breite SIMS-Daten mit hochauflösender NAA. lenr-canr hat eine Reihe von Links zu Artikeln, in denen SIMS und NAA erwähnt werden, und ein kurzer Blick zeigt, dass zumindest einige ziemlich breite Diagramme verfügbar sind. Ich muss noch einen geeigneten Datensatz finden, der ohne allzu große Probleme analysiert werden kann, aber ich hoffe, dass diese irgendwo verfügbar sein werden. Dieses Papier legt nahe, dass Pd-108-Kerne in leichtere Elemente spalten: bit.ly/vm6z4e .
@EricWalker: Wenn Sie noch hier sind, verstehe ich die Transmutationen vollständig . Ich kann Ihnen sagen, welche Experimente gut und welche schlecht sind, eines nach dem anderen.
Woher kommen im klassischen F&P-Experiment diese K-Shell-Löcher und warum erscheinen sie anscheinend die meiste Zeit nicht (oder tun nichts)? Und warum werden dann emittierte Röntgenstrahlen so selten nachgewiesen?
@KirkShanahan: Die K-Shell-Löcher müssen gesät werden, um die Reaktion zu starten, entweder durch kosmische Strahlung, durch eine natürliche Radioaktivität irgendwo, sie können dann entweder durch Röntgenstrahlen zerfallen oder in ein niedrigeres Band gehen. Die oberen Bänder sind durch Fusion multiplikativ instabil, sie werden durch die durch die Fusion erzeugten MeV-geladenen Teilchen wieder aufgefüllt, aber sie benötigen einen Seed. Ich denke, das ist der Hauptgrund, warum die Reaktion sporadisch ist, man braucht radioaktive Kontamination, um sie zu starten. Die Röntgenstrahlen werden nur emittiert, wenn es zu viel Hitze gibt, und es gibt nicht so viele, nur viel pro Fusion. Film erkennt sie regelmäßig.
@KirkShanahan: Gehen Sie zu einem niedrigeren Band, wobei ein Elektron die Energie absorbiert, und nicht ein Röntgenstrahl. Dies ist ein Kompromiss, es ist nicht klar, welcher Effekt dominiert, aber Sie sehen Röntgenstrahlen, die bei der Co-Abscheidung weit über dem Hintergrund emittiert werden (der Kathodennebelfilm), und das allgemeine Spektrum ist KeV-isch (Mosier-Boss). war einer der wichtigsten Hinweise darauf, was los ist.
@KirkShanahan: Es wird angenommen, dass die Röntgenstrahlen kollimiert sind. Unter der Annahme, dass dies zutrifft, könnten sie möglicherweise übersehen werden, wenn sich der Detektor nicht an der richtigen Stelle befindet.
@RonMaimon: Alle Hinweise zu den Transmutationsexperimenten sind willkommen.
@EricWalker: Die Transmutationen sind Kernfragmente des Zuschauer-Pd in ​​der Fusion, die von anderen Pd emittiert und absorbiert werden. Aus diesem Grund sehen Sie Spitzen bei +/- 8 +/- 12 +/- 16 und Spitzen bei jeder +/- geraden Zahl. Leichte Kerne dieser Art zu emittieren ist einfach, dies sind die stabilsten Fragmente (dies wird in der neuen Version der Antwort erklärt, bitte lesen Sie die Änderungen). Die Sache, von der ich dachte, dass sie die Theorie zunichte macht – die großen Transmutationen – sind jetzt tatsächlich der beste Beweis dafür. Ich werde etwas Formales darüber schreiben.
@EricWalker: Die Kollimation ist ein bisschen langwierig, aber der einfachste Weg, dies zu sehen, besteht darin, die Kathode mit Film zu umgeben und nach Flecken zu suchen. Ich habe auf den Röntgenbildern keinen Hinweis auf Flecken gesehen. Ich bin mir sicher, dass während der ganzen Zeit, in der überschüssige Wärme vorhanden ist, Röntgenemissionen auf niedrigem Niveau in der Größenordnung von 10 Photonen pro Fusion auftreten. Das sind nicht viele Photonen pro Joule, da es 24 MeV Wärme pro Fusion gibt und nur 240 KeV an Röntgenphotonen pro Fusion, das ist 1 % Leistung in Röntgenphotonen, es könnte bis zu 10 % der überschüssigen Wärme sein (wenn das ganze Band geröntgt wird), aber das ist die extreme Obergrenze. Es sind ein paar Deziwatt.
@RonMaimon: Ich nehme Ihr Wort zu den Energien, die an der Röntgenstrahlung beteiligt sind. Zur Kollimation siehe Karabut, Karabut and Hagelstein, iscmns.org/CMNS/JCMNS-Vol6.pdf , p. 217ff.
Das Problem bei der Verwendung von röntgenempfindlichen Filmen in Kaltfusionsexperimenten, insbesondere in F&P-Zellen, besteht darin, dass Hitze auch Beschlag verursacht. Zellen werden normalerweise sehr lange bei > RT betrieben, und soweit ich mich erinnere, werden die Filme so nah wie möglich gehalten, um so viele Röntgenstrahlen wie möglich einzufangen, wodurch die thermische Exposition maximiert wird. Außerdem würden kollimierte Röntgenstrahlen klare Flecken auf dem Film ergeben, während alle Filme, die ich gesehen habe, amorphe Kleckse zeigen, außer denen von BHARC. Dort sehen sie echte Fleckenmuster, aber dafür gibt es eine einfache Erklärung – sie haben versehentlich belichteten Film verwendet.
@KirkShanahan: Es gibt eine andere Möglichkeit, dass die Röntgenstrahlen an den Fusionspunkten emittiert werden und vom Kristall gebeugt werden, was ich für viel wahrscheinlicher halte. In SPAWAR-Experimenten wird der Film weit genug von der Kopositionsregion entfernt platziert, um die Wärmeeinwirkung unmöglich zu machen. Ich vertraue den sorgfältigen Beobachtungen der Cold-Fusion-Leute mehr als den nachlässigen Spekulationen von Leuten, die sich Gründe ausgedacht haben, um das Phänomen zu leugnen (mit anderen Worten, ich vertraue Ihrem Urteil diesbezüglich nicht, da Sie sich in der Vergangenheit gewaltig geirrt haben und müssen Sie sich noch umkehren).
@Ron - Nein, ich habe mich in dieser Hinsicht noch nicht "monumental geirrt". Sie haben jedoch Unmengen falscher Informationen produziert und greifen auf Rufmord zurück (wie Sie es gerade wieder einmal getan haben) statt auf vernünftige Diskussionen. Wenn Sie denken, dass ich falsch liege, beweisen Sie es. Geben Sie einen Fehler konkret an und erklären Sie, warum er auf mehr als nur Handbewegungen zurückzuführen ist. Nur eine ... das ist alles.
@KirkShanahan: Ok --- nehmen Sie die Dutzende von Tritium-Beobachtungen. Sie sind ohne Betrug einfach unmöglich, da Sie Tritium nicht chemisch herstellen und seinen Nachweis nicht fälschen können (es ist radioaktiv mit einer klaren Signatur). Bocris, Wolf, Bhabha und Los Alamos entdeckten Tritium. Ende der Geschichte. Fertig. Nichts anderes benötigt. Alter, du nervst mich wirklich. Indem Sie über die Beweise, das Tritium, lachen und für den Rest unsinnige Erklärungen erfinden, machen Sie es unmöglich, eine rationale Diskussion zu führen. Ich werde es nicht länger versuchen, ich habe zu tun.
Alter, ich ignoriere nichts, ich weigere mich nur, blind an eine neue Physik zu glauben, wenn ich weiß , dass Kontamination, Interferenzen und schlechte Analysetechniken solche Zahlen mit Leichtigkeit durcheinander bringen können. Ich brauche mehr Beweise als nur einen Haufen wild variierender Zahlen. Und ich sehe Beweise dafür! Ich will Kontrolle sehen. Wenn sie jemals die Wirkung kontrollieren und vorhersagen können, welche T sie unter welchen Bedingungen bekommen, dann haben sie etwas getan. Das ist NICHT, wo wir heute sind, und sie hatten fast 25 Jahre Zeit, um keine Fortschritte zu machen. Es deutet darauf hin, dass ihre Prämissen falsch sind. Zeit für einen neuen Baum.
@Ron: Bitte poste einen Link zu deinem Artikel, wenn ein Entwurf verfügbar ist.
@EricWalker: Ich werde versuchen, etwas über Thanksgiving aufzuschreiben --- Ich mache einige Bioinformatik-Arbeiten, und das ist sehr zeitaufwändig, da es eine Menge Programmierung erfordert. Es fällt mir auch schwer, gute Produktdaten für 20 MeV Elektron / Photon auf Pd zu finden, obwohl ich die allgemeinen Auswurfmuster gefunden habe, habe ich keine quantitativen Fragmentierungsverhältnisse. Das ist ein wenig nervig, es gibt viele Daten, es ist nur schwer, sie zu durchsuchen. Es ist schwierig, Fragmentierungsverhältnisse theoretisch vorherzusagen.
@RonMaimon: Zwei interessante Kommentare zu deinem Vorschlag: bit.ly/ThCPfj und bit.ly/SgBOqi .
@EricWalker: Spaandonk hat eine falsche Intuition: Die von einem 24 MeV Alpha deponierte Energie wird nicht schnell von Valenzelektronen aufgezehrt, sie wird bevorzugt in K-Schalen deponiert (dies wird durch die Bethe-Formel Ionisation beschrieben). Das Ergebnis ist kein "Klebstoff", geladene Teilchen dieser Energie gehen durch einen mm Metall, bevor sie anhalten, und hinterlassen einen mm von Hunderten von K-Schalen-ionisierten Atomen. Diese K-Schale wird mit D im Metall gemischt und ergibt eine Band. Die 20-KeV-K-Schale kann einen Pd-Kern nicht ausschalten, da ein solcher Kern vollständig ionisiert wäre, und dies ist mehr Energie als nur die K-Schalen-Ionisation.
... diese beiden Dinge waren die ersten Dinge, die ich vor Jahren überprüft habe, und ich habe den Multiplikationsfaktor gefunden, der für verdünnte Fusionsexperimente in Strahlen erforderlich ist, und er war ein paar Größenordnungen daneben (wie 2 oder 3, nicht 10). Das war nicht entmutigend, weil andere Theorien um 10 Größenordnungen daneben liegen und auch das Fehlen von Neutronen bedeutet, dass etwas anderes passiert. Die Doppel-d-Reaktion erfordert, dass das K-Schalenband die Deuteronen zu Unebenheiten auf der Oberfläche leitet, wo sie ausreichend konzentriert sind, um eine Fusion zu ermöglichen. Das Ni-H (falls vorhanden) erfordert, dass die dp-Reaktion stattfindet.
UPDATE (per E-Mail): Ron sieht jetzt, dass die Hagelstein-Bindung eine inkohärente Reaktion verbietet. Dies schließt den inkohärenten Prozess aus, den er als Erklärung für die Exzesswärmeexperimente vorgeschlagen hat, die beobachtete Strahlung ist zu gering. Dies weist auf kohärente Prozesse hin, und er untersucht jetzt kohärente Versionen desselben Prozesses, um Hagelsteins Modell zu vervollständigen, das seiner Meinung nach völlig solide ist und das Phänomen erklären sollte, wenn die geeigneten Modi identifiziert werden.
@Jerry Schirmer: Wenn dieses Modell richtig ist, obwohl es physikalisch interessant ist, ist es möglicherweise nie wirtschaftlich rentabel.
@ Joshua: gut. Aber Tokamaks sind nicht wirtschaftlich, aber <i>sie können und wurden gebaut</i>.
@RonMaimon ist das nicht eine Art kalte Fusion? www1.grc.nasa.gov/space/science/lattice-confinement-fusion

Das wurde sofort auf der APS-Session 1989 in New York wunderbar theoretisch beantwortet, glaube ich, von Koonin. Theoretisch muss man für jede Art von Fusion die Coulomb-Abstoßung der relevanten Kerne in der Größenordnung von MeV überwinden, damit die Kerne nahe genug kommen können, damit sich ihre Wellenfunktionen überlappen und verschmelzen. Aufgrund des Phänomens des quantenmechanischen Tunnelns kann dies auf einige zehn bis hundert keV reduziert werden. Daher sind Temperaturen von >> 10 ^ 5 K oder kalte Myonen (die Elektronen um das 200-fache überwiegen) erforderlich, um den Abstand zwischen den Kernen zu verringern (wie bei der durch Myonen katalysierten kalten Fusion, einem echten Phänomen), oder es ist ein anderer spezieller Mechanismus erforderlich, um dies zu ermöglichen enge Annäherung.

Damit aber überhaupt eine chemisch katalysierte Fusion, also über die Valenzelektronen, stattfinden kann, müsste die Bindungsenergie der beiden H-Atome an den Katalysator so hoch sein, dass die jeweilige Konfiguration der niederenergetischen Valenzelektronen usw ... wären für das Problem notwendigerweise völlig irrelevant, dh unabhängig von ihrer Anordnung könnten sie unmöglich die fusionierbaren Kerne katalysieren, sich nahe genug zu nähern, um zu fusionieren. Keine clevere Packungsanordnung, Quasiteilchen, spezielle Adsorption, spezielle Kristallgitterstrukturen usw. könnten diese Schlussfolgerung jemals ändern. Was auch immer auf solch niedrigen Energieskalen passierte, würde im Vergleich zur Energieskala des für die Fusion erforderlichen Abstands zwischen den Kernen als eine Art irrelevanter Flaum erscheinen.

Daher würde die durch Valenzelektronen katalysierte kalte Fusion gegen die Grundgesetze der Quantenmechanik, Kernphysik usw. verstoßen. Leggett und Baym veröffentlichten etwa zur gleichen Zeit auch ein Argument wie dieses ( hier kostenlos zusammengefasst ). Koonin und Nauenberg haben hier eine genaue Berechnung veröffentlicht , die zeigt, dass eine chemisch katalysierte Fusion funktionieren könnte, wenn die Masse des Elektrons 5-10 Mal größer wäre als sie wirklich ist. Beachten Sie jedoch, dass die Reaktionsgeschwindigkeit sehr, sehr stark von der Elektronenmasse abhängt, so dass dies in unserem Universum unmöglich bleibt.

10 kev funktioniert wegen des Tunnelns.
Gibt es hier eine Verwechslung zwischen Coulomb-Barriere und Bindungsenergie?
@Anna, die Coulomb-Barriere ist die Mindestenergie, die die Kerne überschreiten müssen, um zu "verschmelzen". Dies ist im Sinne einer chemischen Reaktion die Aktivierungsenergie.
Julian Schwinger versuchte, Arbeiten zur Kalten Fusion einzureichen, die jedoch von den Gutachtern abgelehnt wurden. Sie könnten eine interessante Lektüre gewesen sein, da Julian wahrscheinlich Robert Oppenheimers klügster Schüler war.
Da dies auf der ersten Seite erneut auftauchte, werde ich den Kommentar hinzufügen, den ich auch in meiner Antwort mache: Kristalle können sich anders verhalten, als uns die thermodynamische und quantenmechanische Intuition in großen Mengen sagt. Es könnte eine Möglichkeit geben, wie sich ein "virtuelles Elektron" über das Kristallgitter ausbreiten kann, das die von Ihnen erwähnte höhere Masse aufweist. Das Problem bei der Kalten Fusion ist nicht, dass sie theoretisch unmöglich ist, sondern dass sie in Experimenten nicht konsequent nachgewiesen wurde. Sonst gäbe es Maschinen zu kaufen, so wie es Sonnenkollektoren gibt. Wenn die Maschine existiert, kommt die Theorie.
-1 Diese Antwort ist kurzsichtig. Experimentelle Daten sind zu bewerten, indem theoretische Vorurteile beiseite geschoben werden. Es gibt solide theoretische Argumente für KeV-Deuteronen in der kalten Fusion, aber selbst ohne diese Beweise ist das Argument "hohe Energie erfordert hohe Temperatur" einfach falsch.
"Keine clevere Packungsanordnung, Quasiteilchen, spezielle Adsorption, spezielle Kristallgitterstrukturen usw. könnten diese Schlussfolgerung jemals ändern." Perfekt, aber alles beruht auf der Tatsache, dass das Konzept der Valenzelektronen eine Annäherung ist, die in einer Vielzahl chemischer Verbindungen unter adiabatischen Bedingungen gültig ist. Wenn etwas im Mehrelektronensystem die Beschreibung ungültig machen würde, wäre diese Schlussfolgerung ebenfalls ungültig.
Ich erinnere mich, dass die Quasikristalle vor ein paar Jahren theoretisch unmöglich waren und dass der Nobelpreisträger von 2011, Shechtman, unter vielen Vorurteilen durch vorgefasste Vorstellungen von „Unmöglichkeit“ litt (er berichtete dies in einem Videointerview). Theorien sind vorläufig und müssen durch andere, vollkommenere ersetzt werden. Ich werde warten und ich hoffe, dass es zum Wohle von uns allen wahr ist. Normalerweise sind Theorien vorläufig und folgen den Experimenten und nicht umgekehrt.
@annav: Ich glaube nicht, dass ein Mangel an konsistenter Demonstration in Experimenten viel über die Realität eines angeblichen experimentellen Phänomens aussagt. Manche Dinge sind schwer festzumachen. Es hat zum Beispiel Jahre gedauert, um herauszufinden, wie man zuverlässige Transistoren herstellt.
@sigoldberg1: Sie nähern sich der Frage der kalten Fusion aus einem theoretischen Blickwinkel - im Grunde sagen Sie, dass die Theorie dies nicht zulässt. Ich finde es sehr gut, darauf hinzuweisen. Aber im Kern sind die zentralen Behauptungen in Bezug auf die Kalte Fusion zum gegenwärtigen Zeitpunkt empirische. Aus diesem Grund scheint mir, dass Kritik die angeblichen experimentellen Phänomene ansprechen muss und dass das Argumentieren von der Theorie aus die Dinge rückwärts macht; Um Ihnen zuzustimmen, müssen wir davon ausgehen, dass die gegenwärtige Theorie auf diesem Gebiet stichhaltig ist.
@EricWalker Ich stimme zu, und deshalb bin ich offen dafür, überzeugt zu sein, sobald Experimente überzeugend werden.
Es wurde nie chemisch katalysiert. Die Untersuchung begann mit einem wirklich einfachen Fehler, bei dem es um ein stark vereinfachtes elektrisches Feld ging, das von Metallzylindern erzeugt wurde. (Die einfache Gleichung ist innerhalb der Zylinder nicht gültig, wurde aber so angewendet, als ob sie es wäre.)
Wenn die zu kleine Elektronenmasse das Problem ist, ist es dann möglich, dass myonische Atomfusion erreichbar ist?

Das Gerät von Fleischmann und Pons stützte sich auf Kalorimetrie (Messung der Energiebilanz in Form von Wärme), die über mehrere Tage hinweg aufrechterhalten wurde, um festzustellen, dass etwas Unerwartetes in der Zelle passierte.

Dies ist experimentell schwierig, da es hochpräzise Temperaturmessungen erfordert, die gegen eine konsistente Referenz aufrechterhalten werden müssen, und auf einem Verständnis davon beruht, wie das Kalorimeter möglicherweise Wärme durch nicht überwachte Kanäle verliert oder nicht.

Kurz gesagt wird ein großer Wärmeverlust von einer großen Wärmezufuhr subtrahiert, und beide Messungen weisen eine gewisse Unsicherheit auf. Das ist ein Warnzeichen in jedem Experiment, aber kein Todesstoß, wenn die Unsicherheit ausreichend genau quantifiziert werden kann.

Die Situation wird durch den von Fleischmann und Pons berichteten zeitabhängigen Wärmeverlust kompliziert. Wenn dies wirklich der Fall ist, würde dies auf einen unerwarteten Prozess hindeuten, obwohl wir a priori nicht wissen können, ob es sich um eine Fusion oder einen Energiespeicher- und -freisetzungsmechanismus handelt.

Es scheint ein Mangel an Konsistenz in Berichten über Energiegewinn oder Neutronen gewesen zu sein, die die Meinung gegen jede Fusion schwankten, die tatsächlich in der F&P-Zelle stattfindet.


Übrigens – Leute, die noch ein paar Jahre daran festhielten, nachdem sich der Konsens dagegen gewendet hatte, beschäftigten sich mit guter Wissenschaft. Es gab immer eine kleine Möglichkeit, dass der Prozess von einem nicht gemessenen und nicht kontrollierten Faktor abhängig war. Um das herauszufinden, müsste man einen nicht trivialen Satz von funktionierenden Zellen anhäufen und dann untersuchen, wie sie sich von nicht funktionierenden Zellen unterscheiden.

Aber wie der Mann sagte: „Wenn es dir beim ersten Mal nicht gelingt, versuche es noch einmal. Dann hör auf. Früher oder später muss man einfach aufgeben.

der Mann sagte "versuche es und versuche es, bis du Erfolg hast", oder zumindest meine Mutter tat es :) Unabhängig von den Fehlern oder Erfindungen von Pons-Fleischmann oder Rossi-Focardi können wir als Physiker nicht alle Hoffnung aufgeben, einen physikalisch gangbaren Weg zu finden Fusion, sei es kalt oder warm, was eleganter und effizienter ist als die herkömmlich akzeptierten Methoden des Plasmaeinschlusses.
-1 Diese Antwort kennt keine Kalorimetrie. Der Effekt, den Pons und Fleischmann beobachteten, war eine Energieabgabe von 30 % über der Leichtwasserkontrolle, viele, viele Sigma über ihrem (kalibrierten) kalorimetrischen Fehler, zeitlich sporadisch und zu lange anhaltend, um Chemie zu sein. Sofern Sie nicht glauben, dass schweres Wasser magisch ist, muss jede Speicherung und Freisetzung von Energie in der Pd-Elektrode gespeichert werden und ist auf 1 eV pro Atom in einem Gramm Pd beschränkt, eine Grenze, die um ein Vielfaches überschritten wurde. Diese Erklärungen sind so dumm, dass es unverantwortlich ist, sie zu wiederholen.
@Ron Maimon: "Diese Erklärungen sind so dumm, es ist unverantwortlich, sie zu wiederholen." Es gibt die offensichtliche Erklärung, dass Pons und Fleischmann inkompetent waren. Diese Erklärung wird durch die Tatsache gestützt, dass ihre Ergebnisse nicht reproduzierbar sind. Als Nuklearphysiker, der Neutronennachweise durchgeführt hat, ist es für mich auch schmerzlich offensichtlich, dass sie für den Neutronennachweis nicht kompetent waren.
Es scheint, dass Palladium/Deuterium dann als Inkompetenten-Magnet verwendet werden kann, weil die Inkompetenten Paneth/Peters (1924!) + Filimonenko (1956!) + Pons/Fleischmann (1989) nicht zu vermeiden scheinen. Selbst wenn ich die Reproduktionen seither ignoriere und mich in die Schuhe von 1989 versetze, besteht keine Chance, dass dies eine Fälschung ist. Du solltest dich schämen.
@BenCrowell: Ich verstehe, dass die Neutronenerkennung ebenso eine Kunst wie eine Wissenschaft ist. Fleischmann und Pons waren Chemiker. Ich kann mir nicht vorstellen, dass sie dachten, sie seien gut im Neutronennachweis, und sie wussten sicherlich nicht, worauf sie sich einließen. Sie hätten sich Hilfe suchen sollen, und es war ein großer Fehler, es alleine zu versuchen. Sie versuchten 1989, ihre Apparatur nach Harwell zu schicken, wo ausgefeilte Messungen durchgeführt werden konnten, aber der Zoll verhinderte dies, bevor alles um sie herum zusammenbrach. Weitere Informationen finden Sie unter Schließen.
Die Kalorimetrie von F&P war nicht fehlerfrei. In meinem Whitepaper (auf das an anderer Stelle in diesem Forum verwiesen wird) gehe ich mehrere Probleme damit durch, und ich behaupte, dass das Ignorieren oder Nichterkennen dieser Probleme dazu führte, dass F&P zu viel Vertrauen in ihre Kalorimetrie setzte. Und ich bin sicher, dass sie zunächst versuchten, Gammastrahlenemission zu entdecken, dachten, sie hätten sie gefunden, aber es fehlte der Compton-Rand, was zeigte, dass sie die Spektroskopie nicht richtig gemacht hatten, und sie zogen diese spezifische Behauptung zurück.

Pons und Fleischmann berichteten ursprünglich 1989, dass ihre chemischen Zellen überschüssige Wärme, Neutronen und Tritium produziert hätten. Ihre Interpretation war, dass Deuteriumkerne fusionierten, um 4He zu produzieren. Die Verzweigungsverhältnisse in diesem Verfahren sind bekannt: 50 % n+3He, 50 % p+3H und 10^-6 4He+gamma. Wenn die behauptete überschüssige Wärme durch Fusion erzeugt worden wäre, wären die Experimentatoren durch die Neutronen getötet worden, die von den 50% der Zerfälle stammen, die durch Neutronenemission erfolgten. Neutronendetektoren sind unter Kernphysikern berüchtigt dafür, dass sie schwierig zu handhaben sind und dazu neigen, Störsignale zu erzeugen. Selbst wenn alle von Pons und Fleischmann behaupteten Neutronenzahlen real gewesen wären, wäre der beobachtete Neutronenfluss im Verhältnis zu der behaupteten Menge an überschüssiger Wärme um viele Größenordnungen zu klein gewesen.

Seit 2010 besteht unter Wissenschaftlern Einigkeit darüber, dass die kalte Fusion ein Beispiel für die pathologische Wissenschaft war. Einige wenige wahre Gläubige führen jedoch weiterhin Experimente durch und behaupten positive Ergebnisse. Eine Rezension aus dem Jahr 2010 von einem prominenten Gläubigen [Storms 2010] besagt, dass "viele Menschen glauben, dass die Korrelation zwischen Hitze und Helium der stärkste Beweis für die kalte Fusion ist." Das Problem ist, dass Wärme in jedem Fall durch chemische Reaktionen erzeugt wird und die angegebenen Heliumwerte nicht hoch genug sind, um einen überzeugenden Überschuss im Verhältnis zum Hintergrund zu zeigen. Wenn diese Behauptungen richtig wären, würden sie auch eine grundlegende Umschreibung der Gesetze der Physik erfordern. Sie würden erfordern, dass das Verzweigungsverhältnis bei der dd-Fusion durch die chemische Umgebung drastisch verändert wird, aber das ist unmöglich, weil die Elektronen bei einer Kernreaktion nur Zuschauer sind. Um Energie und Impuls zu erhalten, erfordert die dd-Fusion auch die Emission von zwei Teilchen im Endzustand. Um die Nichtexistenz des zweiten Teilchens zu umgehen, nehmen Enthusiasten der kalten Fusion an, dass Energie aus der Reaktion auf das elektronische Gitter übertragen wird. Es existiert kein bekannter Mechanismus, durch den eine solche Übertragung erfolgen könnte. Als die experimentellen und theoretischen Einschränkungen strenger geworden sind, haben die Gläubigen darauf reagiert, indem sie verrücktere Pseudowissenschaften entwickelt haben, einschließlich der Produktion von Kernen mit der Ordnungszahl 126 und der Umwandlung der Elemente durch Pflanzen und Bakterien. [Storms 2010] Enthusiasten der kalten Fusion nehmen an, dass Energie aus der Reaktion auf das elektronische Gitter übertragen wird. Es existiert kein bekannter Mechanismus, durch den eine solche Übertragung erfolgen könnte. Als die experimentellen und theoretischen Einschränkungen strenger geworden sind, haben die Gläubigen darauf reagiert, indem sie verrücktere Pseudowissenschaften entwickelt haben, einschließlich der Produktion von Kernen mit der Ordnungszahl 126 und der Umwandlung der Elemente durch Pflanzen und Bakterien. [Storms 2010] Enthusiasten der kalten Fusion nehmen an, dass Energie aus der Reaktion auf das elektronische Gitter übertragen wird. Es existiert kein bekannter Mechanismus, durch den eine solche Übertragung erfolgen könnte. Als die experimentellen und theoretischen Einschränkungen strenger geworden sind, haben die Gläubigen reagiert, indem sie verrücktere Pseudowissenschaften entwickelt haben, einschließlich der Produktion von Kernen mit der Ordnungszahl 126 und der Umwandlung der Elemente durch Pflanzen und Bakterien. [Storms 2010]

Zusammenfassend lässt sich sagen, dass Behauptungen über die kalte Fusion nicht richtig sein können, es sei denn, sie stürzen fest etablierte Kenntnisse der Kernphysik um. Dies wäre eine außergewöhnliche Behauptung, und es würde einen außergewöhnlichen Beweis erfordern. Nach zwei Jahrzehnten ist kein solch außergewöhnlicher Beweis aufgetaucht.

Gai et al., "Obergrenzen für Neutronen- und Gammastrahlenemission aus kalter Fusion", Nature 340 (1989) 29–34.

Storms, "Status of Cold Fusion (2010)", Naturwissenschaften (online) 97 (10): 861–881

Ihre Antwort ist ziemlich besonnen, aber ich empfehle Ihnen, sich SPAWAR-Experimente und Arata-Experimente anzusehen. SPAWAR zeigt schlüssig, dass es energiegeladene Teilchen gibt, während Arata zeigt, dass Wärme und Helium korreliert sind. Die Heliumkonzentrationen in mehreren Experimenten vom Pons/Fleischmann-Typ liegen weit über dem Hintergrund, und die Tritiumproduktion ist völlig unerklärlich. Was die Übertragung von Kernenergie auf das Gitter betrifft, so ist dies ein Rätsel, das im Prinzip durch eine Dreikörper-Deuteron-Deuteron-Kern-Wechselwirkung gelöst werden könnte, so dass es den Prozess nicht ausschließt.
Das von Ihnen zitierte Gai-Papier: nature.com/nature/journal/v340/n6228/abs/340029a0.html führt keine Kalorimetrie durch. Woher sollten sie wissen, wann die Reaktion stattfand, wenn überhaupt? Vielleicht liefen sie Blindgänger. Vielleicht liefen sie mit zu geringer Belastung. Es ist ein Junk-Paper, und es hätte niemals veröffentlicht werden dürfen.
@Ron Maimon: Wir sind uns offensichtlich völlig uneins darüber, was Garbage Science ist und was nicht. "Vielleicht haben sie Blindgänger gefahren." Natürlich liefen sie Blindgänger. Kalte Fusion existiert nicht, also sind alle kalten Fusionszellen Blindgänger.
Wenn sie den Effekt nicht reproduzieren konnten, welchen Wert hat er, um die Anzahl der Hintergrund-Neutronen zu messen? Sie wollten nur ihre Stimmen auf die „me too“-Liste der gescheiterten Reproduktionen setzen. Ich zeigte meinem sanftmütigen Vater Papiere wie diese und sagte ihm: "Weißt du, solche Leute sollten gefeuert werden." Er sagte: "Solche Leute sollten erschossen werden."
Für die Humorgestörten ist die obige wahre Anekdote ein Witz, keine Drohung.

Es gibt ein paar Gründe.

  1. Es gab nie einen klaren Grund, warum elektrisierendes Palladium Druck erzeugen sollte, der ausreicht, um eine Fusionsreaktion zu zünden. Ohne einen Mechanismus scheint dies die lächerlich radikalste und sensationellste Schlussfolgerung zu sein, die möglich ist, auch wenn die Kalorimetrie besagt, dass elektrifiziertes Palladium irgendwie Nettoenergie erzeugt. Warum nicht mit einfacheren Erklärungen beginnen, als zu sagen, dass es Fusion ist?

  2. der einzige Beweis, den jemals jemand erbracht hat, war die Kalorimetrie eines Experiments im kleinen Maßstab. Aber Kalorimetrie ist eine komplizierte Sache – Sie müssen die Isolierung des Systems genau richtig modellieren, und Sie müssen die Wärmezufuhr für das System korrekt messen und so weiter.

  3. Es wurden noch nie Fusionsprodukte beobachtet - eine Fusion wäre plausibel, wenn sie zeigen könnten, dass die Reaktionen etwas Helium-3 oder Lithium oder was auch immer produzierten.

  4. Der Aufbau, den sie in diesen Experimenten beschreiben, scheint ziemlich einfach zu sein – tauchen Sie einfach etwas Palladium unter Wasser und lassen Sie Strom durchfließen. Wenn dies wirklich eine Fusionsreaktion hervorruft, warum nicht einfach vergrößern und einen Reaktor bauen? Ich bin mir sicher, wenn sie ein Gebäude mit einem Palladium-Fusionsreaktor (oder auch nur einer Glühbirne) mit Strom versorgen könnten, würde jeder Kritiker sofort die Klappe halten und sich hinter sie stellen. Und sie scheinen nie zu erklären, warum sie das nicht einfach tun. Ich werde niemals an Ihre revolutionäre neue Form billiger Energie glauben, es sei denn, Sie beginnen damit, billige Energie zu produzieren (oder zumindest zu erklären, warum Sie das nicht können).

""Elektrisierendes Palladium"", das ist nicht richtig. Sie elektrolysierten eine wässrige Lösung unter Verwendung einer Palladiom-Kathode. Palladium (und einige andere Metalle der Platingruppe) bilden mit Wasserstoff eine Art Verbindung. Wasserstoff wird in Atome gespalten und diese Atome diffundieren durch das Palladium. (Dieser Effekt ist die Grundlage für die Verwendung von Pd als Hydrierungskatalysator.) Dies ist der Hintergrund für die Annahme/Hoffnung, dass es zu einer gewissen "Verdichtung" kommen könnte
@Gerog: Dein Kommentar ist abgeschnitten, aber fair genug. Ich habe nur auf der populären Ebene darüber gelesen, wo es so aussah, als gäbe es einen Schaltkreis aus Palladium, der dann untergetaucht wurde. Dennoch ist jeder chemische Mechanismus, der eine Fusion verursacht, etwas skizzenhaft, wenn man bedenkt, dass die chemischen Energien so viel niedriger sind als die nuklearen - ich würde erwarten, dass die Pd-H-Bindungen brechen, bevor der Wasserstoff genug komprimiert wird, um zu fusionieren. Und trotzdem beantwortet das nicht die Frage, warum sie nicht einfach den verdammten Reaktor bauen.
Hallo Jerry, diese Frage ist einfach: Es geht nicht! Abgesehen von allen Problemen bei diesem Experiment sollte es nach einiger Zeit eine stationäre Situation geben, bei der mehr Energie herauskommt als zugeführt wird. Ich bin sicher, sie haben sich Mühe gegeben, vielleicht für einige Wochen ....
-1 Das Experiment ist sehr schwierig, da es eine 1:1-Ladung von Deuterium in das Pd erfordert, und die Wirkung sporadisch und metallurgisch empfindlich ist. Die Kalorimetrie hingegen ist trivial, da man eine Leichtwasserkontrolle fahren kann und seine Fehler kennt. Kalorimetrie ist eine 100 Jahre alte zuverlässige Wissenschaft.
Tatsächlich gibt es veröffentlichte Berichte über kalte Fusion in Platin, das keinen Wasserstoff absorbiert, was die Idee zunichte macht, dass man eine hohe Wasserstoffbeladung benötigt, um eine kalte Fusion zu erreichen. Gleichermaßen ist bekannt, dass Ni keine nennenswerte Menge an Wasserstoff lädt, bis viele Gigapascal des angelegten Drucks betragen. Auch frühere Forschungen auf diesem Gebiet hatten offensichtlich überschüssige Energie mit Pd-Kathoden, während sie nur eine Beladung von 0,78 H/Pd behaupteten. Der H/M muss 1 Behauptung entsprechen, die aus der Arbeit von McKubre stammt, aber wie bereits erwähnt, widerspricht dies früheren und verwandten Erfahrungen.
@KirkShanahan: Es ist wahrscheinlich, dass sich das Pt-Zeug in Oberflächenschichten befindet und das Ni alles Rekombinationsfehler ist (vielleicht auch Pt). Der Grund, warum man McKubre besondere Aufmerksamkeit schenkt, ist, dass er zuverlässig nuklearen Mist, He, erhält und darauf achtet, das He zu quantifizieren und auf Rekombination zu prüfen. In der Theorie, die ich gegeben habe, ist unbedingt eine hohe Bulk-Beladung erforderlich, um nukleare Effekte zu beobachten. Sie benötigen homogene 1-1-Abschnitte, um eine gute d-Bandbildung ohne Streuung zu erhalten. Soweit es mich betrifft, sind alle Beobachtungen von überschüssiger Wärme in Ni- H oder in Pt sollten vor der Annahme mit Tritium und Transmutationen bestätigt werden.
@KirkShanahan: Der Effekt, falls vorhanden, wird oft eher als Oberflächen- als als Masseneffekt angesehen. Zwar versuchten Fleischmann und Pons, die Palladiumkathode mit Deuterium aufzuladen, und hatten dafür eine Reihe von Gründen, die einen Masseneffekt vermuten ließen. Aber das könnte ein Ablenkungsmanöver gewesen sein. Es ist also möglich, dass die Durchlässigkeit irrelevant ist.
-1 Es gibt viele Berichte über „nukleare Asche“ – Helium-4, Tritium, niedrige Neutronenkonzentrationen über dem Hintergrund, schnelle Alpha-Teilchen und Protonen sowie Röntgenstrahlen. Es kann sein, dass diese Berichte alle falsch oder unzureichend sind. Aber wir sollten die Fragwürdigkeit der Berichte ansprechen, anstatt wie in (3) oben eine pauschale Behauptung aufzustellen, dass nie Fusionsprodukte gefunden wurden.
@Ron - Nun, zumindest hast du die Hälfte meiner Punkte bekommen. Ja, die Debatte drehte sich immer um „Oberfläche oder Masse“. Die Pt- und Ni-Arbeiten zeigen, dass es sich um eine Oberfläche handelt. Nur weil Pd-Hydride einfacher sind, sollte man keinen anderen Mechanismus annehmen. Und beachten Sie noch einmal, dass PdHx mit x < 0,8 berichtet wurde, um Effekte zu zeigen, also ist > = 1,0 NICHT erforderlich.
@Eric - Von Anfang an gab es Behauptungen über entdeckte Atomasche, und von Anfang an wurden diese Behauptungen in Frage gestellt. Zwei allgemeine Punkte. (1) Reproduzierbarkeit ist nicht vorhanden, (2) analytische Methoden werden nie erklärt und einige spezifische Ergebnisse von Clarke deuten darauf hin, dass sie es wirklich, wirklich sein müssen. Bis (1) und (2) behandelt werden, sind die Ansprüche nicht zwingend.
@KirkShanahan: Es ist ganz in Ordnung, wenn die Leute die Behauptungen von Fusionsprodukten nicht überzeugend finden. Aber die Leute sollten versuchen, ein Gespräch zu führen und bestimmte Details anzusprechen, anstatt sich auf pauschale Behauptungen zu verlassen, denen das Publikum zustimmen soll (es gab keine Fusionsprodukte, keine Reproduzierbarkeit, keine Erklärung von Analysemethoden). Sie selbst sehen die Notwendigkeit dafür, da Sie an anderer Stelle bewundernswert ins Detail gegangen sind. Harwell, MIT und Caltech sahen die Notwendigkeit dafür.
@Eric - Und der wahre Grund, warum die kalte Fusion als Schwindel angesehen werden sollte, ist, dass die kalten Fusionierer sich weigern, dies zu tun. Von Anfang an wurden ihre He-Ergebnisse als Lecks angezweifelt, aber keiner von ihnen berichtet über die Hintergrundwerte der Laborluft während des Versuchszeitraums. Vor 12 Jahren habe ich ein später veröffentlichtes Manuskript in Umlauf gebracht, das zeigte, wie Wärmesignale auf Watt-Ebene Fehler sein könnten, aber das Fachgebiet hat nicht einmal zugegeben, dass ich das getan habe, geschweige denn die Art und Weise geändert, wie sie Experimente durchführen. Die derzeitige Generation kalter Fusionierer ist einfach zu voreingenommen, um vernünftige Diskussionen darüber zu führen.
In Bezug auf das Laden: Ich bin nicht davon überzeugt, dass alle Experimente mit Ausnahme derer, die Nuklearprodukte sehen, zuverlässig nuklear sind. Ich will Er, und ich will Tritium, und noch besser, Transmutationen. Palladiumdeuteride, die mit 0,85 beladen sind, werden an der Oberfläche vollständig beladen, das Deuterium diffundiert ein, und jeder vollständig deuterierte Abschnitt, selbst wenn er klein ist, ist möglicherweise ein Ort, an dem Sie eine Fusion haben können. Das Oberflächenproblem ist nicht so wichtig, aber das Protonenproblem ist: Auf keinen Fall sind die Protonenexperimente zuverlässig nuklear, Sie können die Coulomb-Barriere nicht überwinden. Aber diese sehen keine Asche.
@EricWalker: Ich denke nicht, dass es in Ordnung ist, die Behauptungen von Fusionsprodukten nicht überzeugend zu finden: Das ist nicht vernünftig, da Bocris, Wolf, die Bhabha-Leute, Mizuni, Iwamura und ein Dutzend anderer Leute, die Tritium gefunden haben, dazu erforderlich sind regelrechten Betrug begangen haben. Es gibt nur zwei Möglichkeiten: Bocris hat Betrug begangen, oder Cold Fusion passiert. Es gibt keine dritte Option. Aus diesem Grund wurde Bocris wegen ethischer Verstöße zur Rechenschaft gezogen – seine Ergebnisse sind ohne kalte Fusion unmöglich . Da ist meiner Meinung nach kein weiteres Gespräch nötig, das Leugnen ist an dieser Stelle alles Politik und keine Wissenschaft.
Nein Ron, niemand hat berichtet, dass er meines Wissens 10x atmosphärisch ist. Das Beste ist ~ 2x, wie ich mich erinnere. Aber atmosp. Ebenen sind nicht relevant. Die Expt. wurden in Laboren betrieben, und Laborluft kann ihn von dem Typen nebenan enthalten, der Leckprüfungen durchführt! Und wieder nennst du mich unehrlich. Wie? Die Daten, die ich analysiert habe, habe ich unter der Annahme gemacht, dass es keine Übertreibungen gibt, und das habe ich klar gesagt. Der Rest ist nur das direkte Ergebnis der Algebra, keine Entscheidungen von mir, wie Sie an anderer Stelle angedeutet haben. Ich habe mir nur die Ergebnisse angesehen und eine ausführliche herkömmliche Erklärung hinzugefügt. Unehrlich? Ich glaube nicht.
Er in diesen Zellen in den gemeldeten Mengen kann leicht aus Lecks kommen. Jeder , der ähnliche Ergebnisse meldet, fällt unter die gleiche Bewertung. Zeig mir, wie ich das „Produkt“ Er vorhersagbar kontrollieren kann, und ich könnte glauben.
@Ron: Ich bin definitiv mit dieser Ansicht einverstanden. :)
zu: Transmut. - Sehen Sie sich zum Beispiel Rolison (Anal.Chem. 63(1991)1697) an. - eine XPS-Studie von elektrolysiertem Pd in ​​D2O und H2O. Sie finden heraus, dass Massen-M-Verunreinigungen in beiden Fällen an die Oberfläche gewandert sind. Kalte Fusionierer würden Transmutation als Quelle angeben, aber Rolison ist viel ehrlicher. Sie stellen auch fest, dass sie einen Monat nach Beendigung der Verwendung von D2 aus Pd herauskamen, was für SIMS-basierte Behauptungen von Isotopenverschiebungen relevant ist. Die beste Erklärung für Fremdelemente auf der Oberfläche ist die Verunreinigung. Beachten Sie, dass S. Little zeigte, dass das Auslaugen von Zellteilen die Quelle für einige war. Hatten T und He schon.
Das Fazit ist, dass bei jeder Methode, die Sie zur Auswahl der echten CF-Fälle wählen, die tatsächlichen Ergebnisse zu nicht reproduzierbar sind, um alles andere als suggestiv zu sein. Angesichts dessen sind die beobachteten Werte auch konventionell leicht erreichbar, was Ihnen (angeblich) zwei brauchbare Erklärungen für die Ergebnisse lässt, eine konventionelle, eine nukleare. Aber die nukleare erfordert neue Physik, die konventionelle nicht. Die „normale“ Wahl in diesem Fall ist die herkömmliche Erklärung. Die CFler werden das jedoch nicht einmal in Betracht ziehen. Alleine deswegen würde ich ihnen keinen Cent geben.
@KirkShanahan: Du machst dumme Politik. Sie können keine radioaktiven Elemente (erkannt durch Gamma) von einer nicht radioaktiven Kathode erhalten. Keine Segregation kann Tritium produzieren, und ja, die Coposition He-Werte waren so hoch wie das 10-fache der Atmosphäre. Die Fusion erfordert keine neue Physik, ich habe erklärt, wie sie passiert, sie wird von der bestehenden Physik verlangt, und man kann vorhersagen, dass sie regelmäßig in jedem ausreichend schweren Metall stattfindet, das deuterieren kann, und es gibt keine Möglichkeit, sie zu vermeiden. Die Experimente sind ausgezeichnet für ihr Finanzierungsniveau, und Ihre Pseudo-Erklärungen der Ergebnisse sind erbärmlich.
Schöner Fehler Ron. Ich bitte um eine detaillierte, begründete Darstellung nur eines meiner „Fehler“ und bekomme das obige ... PS: Ich glaube Ihrer Theorie nicht. PPS Ich glaube nicht, dass 10X atmosphärisch ist - zitieren Sie Ihre Referenz. bitte. Oh, und PPPS-Lecks werden tatsächlich dazu führen, dass He im Laufe der Zeit zunimmt.
Es könnte sich lohnen, einen kurzen Blick auf einige Zahlen zu werfen. Aus dem Buch von Storms 2007, Tabelle 6, Tritium-Ergebnisse: Bockris et al. 1989 2x10e5 x bkgrd; Inyengar et al (BHARC) 1989 8 x 10e15 --- das ist richtig 10 Größenordnungen Unterschied. Mizun_o_ und Iwamura sind eher für die Transmutation von Schwermetallen bekannt. siehe nächster Kommentar.
Iwamura 2000 Pd/LiOD+D2O F,Al,Si,S,Cl,C,Ca,Ti,Cr,Mn,Fe,Co --- Iwamura 1998 Pd/LiOD+D2O Ti, Cu, Fe (Isotopenwechsel) - -- Mizuno 1996 Pd/LiOH+D2O Pt,Pb --- Rolison 1991 Pd/LiSO4+D2O,H2) Rh,Ag ----- wirklich konsistent, richtig ... der Mangel an Konsistenz deutet wirklich auf eine Kontamination durch verschiedene Ursachen hin Materialien aus verschiedenen Quellen. Und erinnern Sie sich, dass Rolison ihre Elemente Pd-Massenverunreinigungen zuschrieb, die sich auf der Oberfläche konzentrierten
@KirkShanahan: Die Ag-, Ru-Transmutationen sind signifikant, weil sie von Wolf unter Verwendung von Gamma-Emissionen radiologisch nachgewiesen werden. Die Bocris-Ergebnisse waren viele Male Hintergrund und wurden in einem unabhängigen Labor durchgeführt, das genau weiß, was Inyengar getan hat. Ich verlasse mich auf Iwamura-Daten, weil sie sich auf große Transmutationen konzentrierten, ich weiß nichts über Rolison. Ich weiß nur, was die Theorie vorhersagt, sie sagt die Fragmentierung von Protonen, Alphas oder den Ausstoß hochstabiler Kerne (Vollschalen) voraus, die für die 20-MeV-Anregung eines Pd-Kerns geeignet sind, und dies erzeugt Pd-Atome mit einer Masse von 4, 8, 12, 16 Einheiten.
@KirkShanahan: Ich mag es nicht, wie Sie sorgfältige Experimente mit nachlässiger Arbeit vergleichen, Sie können alles diskreditieren, indem Sie mit den schlechtesten Ergebnissen vergleichen. Ich stimme zu, dass es Papiere gibt, die auf diesem Gebiet nicht herausragend sind, wie auf jedem Gebiet, aber man muss nach den am sorgfältigsten analysierten Daten suchen und sich damit auseinandersetzen und sie nicht ablehnen, weil jemand anderes auch ein beschissenes Experiment durchgeführt hat.
Ich bestreite nicht, dass Forscher „neue“ Elemente finden, ich bestreite ihre Quelle. Es besteht keine Notwendigkeit, nuc anzunehmen. rxs., Migration von Kontaminationen löst das Problem gut und wurde an anderer Stelle demonstriert (z. B. Little und Rolison). Welche Bockris-Ergebnisse? Den 10X He hast du in dem anderen Thread vermerkt? Siehe meine Antwort dort, 10X ist nicht real. Eigentlich gleich. Soweit ich gesehen habe, gibt es keine sorgfältigen Experimente, die am Ende nuc beanspruchen. empfangen. Die Vorsichtigen finden, dass es eine Verunreinigung ist.

Diese neue „kalte Fusion“ , über die in Wirklichkeit ein Blog berichtet wird, ist in jeder Hinsicht ein kommerzielles Unternehmen. Ihre Ansprüche sind so groß, dass entweder ihre Konstrukte erfolgreich sein werden oder sie ihren Hut fressen werden. Wir müssen nicht lange warten.

Wenn sie erfolgreich sind, wird die Theorie gefunden.

Eine Anmerkung zu Kristallen (sie verwenden Ni-Kristalle) und großen Energien: Entlang der Kristallachse gehen hochenergetische Myonenstrahlen intakt durch, ohne mit der Coulomb-Barriere zu interagieren. Schwer zu findende Referenzen, ich weiß, dass Tom Ypsilantis in den 80ern daran gearbeitet hat. Hier ist ein Vorschlag für einen Myonenbeschleuniger , der das Konzept verwendet. Wenn die Ankündigung also kein Schlangenwasser ist, wird eine angemessene Festkörper-/Kerntheorie entstehen.

Ich bearbeite dies, um ein aktuelles Video der NASA einzufügen , das möglicherweise besagt, dass die Rossi-Behauptungen möglicherweise doch nicht völlig falsch sind. Beachten Sie das Ni28 in der Tabelle der möglichen Elemente, die für die Kernfusion mit niedriger Energie (LENR) verwendet werden können.

Übrigens planen Rossi et al. jetzt kleine Heizungen ! Laptopgröße für den heimischen Markt!! Wenn es sich um einen Betrug handelt, wird es der Betrug sein, der alle Betrügereien beendet!

Einer der Professoren, die ich schon lange zurück hatte, war Joseph Weber. Er war überzeugt, dass Kristalle einen hohen Wirkungsquerschnitt von Neutrinos einfangen könnten. Ich erinnere mich, dass er die Navy dazu gebracht hat, es zu finanzieren. Er erzielte positive Ergebnisse, die niemand je wiederholte. Damals versuchten Doktoranden im Grunde, ihm auszuweichen. Aber er war ein toller Kerl. Google "joe weber"+Kristall+Neutrino
Dieses NASA-Patent , das „Plasmon-Polaritonen“ und „schwere Elektronen“ erforscht, scheint mit diesem NASA-Video verwandt zu sein.
dieser Artikel des NASA-Chefwissenschaftlers „Etwas Reales passiert … NASA LaRC hat mit LENR-Designstudien begonnen, die von … geleitet wurden“ (gefunden über den 22passi-Blog)

Sehr einfach! Sie können die Ergebnisse niemals auf wissenschaftliche Weise wiederholen, um anderen zu zeigen, dass es funktioniert. Was nützt die Wissenschaft, wenn wir die wissenschaftliche Methode einfach ignorieren? Wenn sie sich wirklich etwas ausgedacht hätten, müssten sie nicht geheim sein und nicht genau zeigen, was sie getan haben. Wenn sie es irgendwie durch Zufall bekommen haben, dann ist das keine Wissenschaft. Sobald sie in der Lage sind, die Ergebnisse zu reproduzieren und anderen zu zeigen, und andere sie reproduzieren können, wird es nützlich und wird zur Wissenschaft. (auch wenn kalte Fusion möglich ist, ist es nutzlos, wenn wir es nie reproduzieren können)

Der Grund, warum kalte Fusion als Schwindel gilt, liegt hauptsächlich darin, dass viele Leute es versucht haben und alle gescheitert sind ... und diejenigen, die behauptet haben, erfolgreich zu sein, haben nie bewiesen, dass sie es geschafft haben.

Niemand weiß, ob kalte Fusion möglich ist, aber angesichts all der Schemata und Nachteile, die in der Vergangenheit passiert sind, ist es einfacher, skeptisch zu sein als nicht. Wenn jemand einen Weg entdeckt, wird es SEHR einfach zu beweisen sein, und wenn sie echte Wissenschaftler sind, werden sie damit keine Probleme haben.

Ich denke, es gibt andere Gründe, die besser sind als dieser. Zum Beispiel hatten die frühen Laser Schwierigkeiten mit der Wiederholbarkeit, bis andere die subtilen Techniken im Zusammenhang mit der Impedanz von Stromversorgungskabeln usw. lernten. Im Allgemeinen ist alles Neue experimentell schwierig, und wenn Sie nicht wissen, wie es funktioniert, haben Sie es Schwierigkeiten beim Wiederholen haben, weil Sie nicht genau wissen, was Sie wiederholen. (Das heißt, ist die Farbe der Farbe von Bedeutung? Könnte es sein, dass eine Chemikalie in der Farbe kritisch ist.)
Ähm, nein, du bist völlig daneben. Ich spreche nicht von der Wiederholbarkeit der Messung, sondern von etwas, das Kenner die wissenschaftliche Methode nennen ... lernen Sie es und lesen Sie noch einmal, was ich gesagt habe.
Die wissenschaftliche Methode (sm) hatte nichts mit den vergangenen wissenschaftlichen Revolutionen von Newton, Einstein, ... zu tun (ich sage nicht, dass sm für die kleinen inkrementellen Schritte irrelevant ist). Intuition und harte Arbeit gegen das Establishment sind die Norm.

Warum wurde die Kalte Fusion als Schwindel angesehen? Weil es nicht einfach reproduziert werden konnte, als es ursprünglich angekündigt wurde, weil der ursprünglich vorgeschlagene Mechanismus mit der damals bekannten Physik nicht übereinstimmte und weil die damals vorgelegten Beweise, die angeblich zeigten, dass es sich um Kernfusion (insbesondere DD-Fusion) handelte, fehlerhaft waren.

Vielleicht ist die bessere Frage: Sollte es heute als falsch angesehen werden? Nein, nicht vollständig. Es gibt viele Beispiele für ähnliche Ergebnisse, aber die Wirkung ist noch nicht unter Kontrolle. Der beste Grund dafür ist, dass die Forscher versuchen, die falschen Dinge zu steuern, wenn sie ihre Experimente durchführen, und das führt zu vielen unterschiedlichen Ergebnissen, je nachdem, welche Werte die wirklichen steuernden Faktoren während des Experiments angenommen haben.

Es gibt herkömmliche Erklärungen für diese halb reproduzierten Ergebnisse, die auf andere zu kontrollierende Dinge hindeuten, die die Forscher der Kalten Fusion jedoch nicht anwenden, weil sie sich weigern, nichtnukleare Lösungen in Betracht zu ziehen. Weitere Einzelheiten finden Sie unter der folgenden URL und lesen Sie sie und die darin enthaltenen Verweise:

https://docs.google.com/open?id=0B3d7yWtb1doPc3otVGFUNDZKUDQ

(siehe http://www.networkworld.com/columnists/2012/102612-backspin.html?page=1 )

-1: Das Fehlen vollständiger Kenntnisse der steuernden Faktoren reicht nicht aus, um einen nuklearen Effekt zu leugnen. Alles, was Sie für einen nuklearen Effekt benötigen, ist eine zuverlässige Messung einer nuklearen Transmutation jeglicher Art. Die Tritiummessungen reichen aus, um dies zu beweisen, es wird nichts weiter benötigt. Man kann nur dann gute Fragen zu den steuernden Faktoren stellen , wenn man anerkennt, dass es in diesen Systemen nukleare Phänomene gibt. Man muss nicht alles verstehen, um etwas zu verstehen (obwohl ich mittlerweile glaube, alles verstanden zu haben). Man kann sicher sein, dass man Tritium nicht durch nichtnukleare Chemie herstellen kann.
Nein, Sie müssen zweifelsfrei sicher sein, dass die von Ihnen gemessenen Materialien nicht auf herkömmliche Weise geliefert wurden. Leider ist dies nie ausreichend gezeigt worden. Die stark variierenden Ergebnisse, die von CFern erzielt werden, implizieren tatsächlich, dass sie die wirklichen Kontrollfaktoren nicht kontrollieren, und nach 25 Jahren mit den gleichen schlechten Ergebnissen ist es an der Zeit, einen anderen Baum zu bellen. Geben Sie uns eine Gleichung, die besagt: "Wenn Sie x Menge von A hinzufügen und z Stunden lang bei y Grad erhitzen, erhalten Sie B auf dem und dem Niveau +/- 10%." um die Frage endlich zu beantworten.
Dies ist keine vernünftige Methode, um ein Phänomen zu bewerten, das sporadisch auftritt und schlecht verstanden wird. Was Sie wollen, ist eine statistische Sache --- Wenn ich 100 verschiedene Chargen von Pd bestelle und sie mit Deuterium belade, werden Sie in 5% der Proben Tritium in Konzentrationen weit über dem vernachlässigbaren Hintergrund nachweisen. Dies hängt von Faktoren ab, die die Leute nicht verstehen. Meiner Meinung nach ist das Wichtigste eine angemessene radiologische Kontamination des Pd, die einen Keim für den Start der Reaktion liefert. Wenn Sie Pd mit einem guten Alphastrahler legieren, können Sie möglicherweise eine 100% reproduzierbare Probe herstellen (obwohl es auch explodieren könnte).
Außerdem habe ich irgendwo bei Ihnen einen kurzen Kommentar zur 115-KeV-Röntgenbestrahlung von protoniertem Pd gesehen (kann ihn nicht wiederfinden), um darauf zu antworten: Die Bande, die ich erwarte, liegt bei etwa 20-40 KeV (ich kann nicht sagen Sie genau wo, weil es sich um gemischte Deuteron- und Innenschalenmoden handelt und es von der Deuteronmasse abhängt, aber auf keinen Fall in der Nähe von 115 KeV liegt). Dies bedeutet, dass Röntgenbestrahlung mit 3-4-mal höheren Frequenzen nutzlos ist. Sie benötigen Röntgenexperimente in der Nähe der Pd-K-Schale, um festzustellen, dass sich die K-Spitze verbreitert und sich in ein Band mit einer Bandkante irgendwo in der Nähe des alten K bewegt -Hülse.
Anspruchsvolle reproduzierbare Daten sind der Eckpfeiler der Wissenschaft. Zuverlässige Daten sind reproduzierbare Daten. Aufgrund möglicher Interferenzen kann man kein einziges, zuverlässiges Ergebnis erhalten, was bedeutet, dass Sie nicht beweisen können, dass es von dem stammt, von dem Sie behaupten, dass es stammt. Sie müssen zeigen, dass die vorgeschlagenen Faktoren mit Daten wichtig sind. Mit genügend Daten kann man von Zuverlässigkeit sprechen. Stark schwankende Zahlen bedeuten, dass Sie einen wichtigen Faktor nicht kontrolliert haben. Um Faktoren zu identifizieren, dürfen vernünftige Kandidaten NICHT willkürlich ignoriert werden. CFer blockieren herkömmliche Faktoren. Die unterschiedlichen Ergebnisse bedeuten, dass das schlecht war.
Übrigens, ein weiterer Kommentar zu dieser Sache mit der Maimon K-Shell-Theorie. Meistens wird kein CF-Effekt beobachtet, es müssen besondere Umstände geschaffen werden, und ich habe noch nie von CFer gehört, die Apparate einer keV-Strahlung aussetzen. Wie bekommt man dann normalerweise CF von diesem K-Shell-Elektron-Ding?
Sie verwechseln Photonen und geladene Teilchen. Photonen müssen abgestimmt werden, geladene Teilchen erzeugen keV-Löcher ohne Abstimmung, dies ist die Bethe-Formel-Ionisation. Die Einschränkungen für ein Band bestehen darin, dass Sie die Löcher ausreichend in einer Region konzentrieren müssen, um eine Fusion zu ermöglichen. Dies kann ein starkes elektrisches Oberflächenfeld erfordern, das die Deuteronen konzentriert, um eine dd-Fusion zu ermöglichen (deren Rate in der d-Dichte quadratisch ist). Sie müssen die Reaktion auch initiieren, also benötigen Sie eine gewisse Menge an Radioaktivität geladener Teilchen auf dem Pd. Dies ist schließlich unvermeidlich, es sammelt sich Crud an der Kathode an.
Es gibt mehrere mögliche Gründe für "besondere Bedingungen", Sie benötigen möglicherweise Spuren von Alpha-Emittern an der Kathode, die von Probe zu Probe oder vom Lösungstyp variieren können, Sie haben möglicherweise unterschiedliche kosmische Strahlung oder Hintergrundradioaktivität, die der Auslöser sein können , oder es könnte an den erforderlichen elektrischen Oberflächenfeldern liegen, um das Deuteron-k-Schalenloch in ausreichender Dichte zu konzentrieren, um die Reaktion zu zünden. Wer weiß. Da Menschen wie Sie vorsätzlich die Recherche zu diesem Thema verhindert haben, kann nur sehr wenig definitiv gesagt werden. Aber ich bin mir jetzt sicher, dass mein Mechanismus richtig ist.
Und warum würde man einen Elektronenstrahl nicht als Strahl geladener Teilchen betrachten? Das Schießen eines 2-3-keV-Elektronenstrahls auf ein Metall erzeugt Auger-Elektronen (und manchmal auch Gammas) in der Nähe von 1-1000 eV, eindeutig keine Resonanzbedingung, aus K- und L-Schalenlöchern und deren Füllung durch Valenzelektronen; eine sehr gut definierte Oberflächenanalysetechnik. Ich denke, Ihre Kriterien und Theorie sind falsch, aber ich gebe zu, dass ich nicht qualifiziert bin, Ihr Problem zu diagnostizieren. du musst dir jemand anderen holen.
Ja --- ein Elektronenstrahl würde funktionieren, wenn er bei 100 KeV-1 MeV wäre, aber vorzugsweise ein kommerzieller LINAC bei 20 MeV, das sollte hervorragend sein, um die kalte Fusion zu stimulieren. Der Auger-Prozess ist das, was ich ausnutze, außer dass ich anmerke, dass Sie Auger-Deuteronen auch in einem deuterierten Metall erhalten, nicht nur Auger-Elektronen, und dass Auger-Deuteronen delokalisiert sind und fusionieren und die Fusion mehr Auger-Deuteronen durch seine K-Schale erzeugt Löcher. Dies ist der Hauptpunkt. Ich bin froh, dass Sie sich damit auseinandersetzen, aber ich ermutige Sie, weiterzulesen und weiter zu kritisieren, da Sie sehen werden, dass es kein Problem mit meiner Idee gibt.

Wenn es einen Fusionsprozess gegeben hätte, hätte es auch Neutronenproduktion gegeben. Es gab einige Behauptungen über Neutronen, aber sie wurden nicht verifiziert. Es gibt ein weiteres Problem mit Neutronen, wenn es eine Fusion gegeben hätte, wären Pons und Fleischmann von Neutronen bestrahlt worden. Sie könnten tatsächlich krank geworden oder gestorben sein. Irgendetwas stimmte an der ganzen Sache von Anfang an nicht. Wenn sie beabsichtigten, eine kalte Fusion zu erreichen, warum haben sie sich dann nicht abgeschirmt? Wenn sie nicht beabsichtigten, eine kalte Fusion zu bekommen, aber später vermuteten, dass sie sie hatten, warum schirmten sie dann nicht gegen Neutronen ab? Sie spielten einige Zeit mit diesem Aufbau herum, und wenn sie wirklich dachten, sie würden eine Fusion bekommen, warum haben sie den Apparat dann nicht hinter Bleiziegeln aufgestellt? Entweder waren sie dumm oder sie haben betrogen.

Die Vorstellung, dass interionische Gitterpotentiale Kerne zusammenzwingen könnten, ist lächerlich. Es besteht die Aussicht auf eine Bosonisierung oder Kondensationsphysik. Hier treten D oder T in denselben Quantenzustand ein und können in H^4 übergehen. Dies ist jedoch unplausibel.

„Warum haben sie den Apparat nicht hinter Bleiklötze gestellt? Entweder waren sie dumm oder sie haben betrogen.
Ich weiß nicht, was die Stoppkraft von Blei für Neutronen ist. Sie scheinen zu implizieren, dass Blei nicht funktioniert. Ein Moderator, vielleicht Graphit, oder etwas, das Neutronen stoppt, wäre jedoch umsichtig, wenn Sie etwas tun, das sie erzeugt.
Neutronen verlieren Energie durch Kontaktstreuung mit Kernen und verlieren bei jeder Streuung einen größeren Teil, wenn der Kern leicht ist. So werden oft Wasser oder Kunststoffe verwendet. Das Hinzufügen von Bor, Chlor, Gadolinium oder etwas anderem mit einem hohen thermischen Einfangquerschnitt kann helfen, erzeugt jedoch Gammas, die mit einer Schicht mit hohem Z abgeschirmt werden müssen. Grundsätzlich sind Neutronen ein Schmerz und der beste Plan ist, einfach Abstand zu halten.
Der klassische Neutronenschutz ist Paraffinwachs, heute könnte PE oder PP mit der gleichen Wirkung verwendet werden. Die Streuung erfolgt durch die Protonen im Wasserstoff des Paraffins. Wasser braucht einen Behälter, Paraffin nicht.

Tatsächlich konnten Wissenschaftler meines Wissens einige der Experimente erfolgreich reproduzieren, sie verstehen den Prozess nur noch nicht vollständig. Es wird geforscht, um eine funktionierende LENR-Theorie (wie sie in der Physik-Community genannt wird) zu entwickeln, um genau zu modellieren, was passiert (unerklärliche übermäßige Wärmeerzeugung). Wie einige gesagt haben, ja, es ist ein sehr komplexer Prozess und es ist immer noch möglich, dass die Energie durch andere Mittel (nicht nuklear) freigesetzt wird, aber ich würde kaum sagen, dass es auf die eine oder andere Weise vollständig bewiesen ist. Eine gute Website, um mehr zu erfahren, ist http://lenr-canr.org/

Zitieren Ihrer guten Website: Es wurden keine Versuche gemeldet, dies von anderen Gruppen zu replizieren
Nein, Kaltschmelzen war noch nie zuverlässig reproduzierbar. Wann immer jemand Kompetentes versucht, es zu reproduzieren, passiert es nicht. Siehe z. B. Gai et al., „Upper limits on neutron and gamma-ray emission from cold fusion“, Nature 340 (1989) 29–34.
@BenCrowell: Gai ist nicht kompetent. Die kompetenten Leute sind diejenigen, die es reproduziert haben.

Update: Ich denke, es ist der gleiche Grund, warum die Multi-Universums-Theorie von einigen als falsch angesehen wird: "fantastische Spekulation, losgelöst von der Realität, auf die wir empirisch zugreifen können"

http://www.scientificamerican.com/blog/post.cfm?id=is-speculation-in-multiverses-as-im-2011-01-28

Alter Inhalt hier unten: Ich wollte dies als Kommentar zu Ihrer Frage senden, aber ich habe nicht genug Repräsentanten, um das zu tun.

In dem Buch "Forbidden Science" wird die Kalte Fusion diskutiert und die Haltung der wissenschaftlichen Gemeinschaft erwähnt. Einen Einblick in die Physik-Community gibt auch das Buch „Der Ärger mit der Physik“.

Anstatt sich zu fragen, warum es als falsch angesehen wird, sollte es vielleicht von einigen in als falsch angesehen geändert werden. Ich konnte keine Bestätigung für die Geschichte finden, dass vor der Erfindung der Flugzeuge "mathematische" Beweise im Umlauf waren, warum es unmöglich war, dass Maschinen aus Metall fliegen konnten. (hat jemand Referenzen dazu?)

Das ist genau der Punkt: Die moderne Physik sagt Ihnen, dass kalte Fusion unmöglich ist. Das Gegenteil zu behaupten bedeutet, entweder die Theorie zu verwerfen, die durch zahlreiche Fakten gestützt wird, oder eine neue Art von Wechselwirkungen zu erfinden, die von allen Physikern mit ihren Teilchenbeschleunigern auf wundersame Weise übersehen wurde und doch hier in Ihrem Glas Wasser passiert.
Verwechseln Sie nicht Mathematik und Physik. In der Physik ist Mathematik ein Werkzeug – möglicherweise ein Werkzeug in einem Aufbau, um Beweise zu erbringen. Mathematik ist jedoch nie zu Ende. Das einzige, was in der Physik zählt, ist empirische Beobachtung, typischerweise durch Wiederholung von Experimenten. Die Physik muss also keine mathematischen Konstruktionen als Beweis dafür liefern, warum etwas unmöglich ist, es ist Sache derjenigen, die behaupten, dass etwas möglich ist , empirische Beweise zu erbringen. Dieser Geschäftszweig mag für einige nicht bequem sein, was für die Physik keine Rolle spielt.
@gigacyan : Lieber Gigacyan, in Sydney gibt es maritime Gebäude, jedes Mal, wenn ich am Eingang des Gebäudes vorbeigehe, sehe ich einen riesigen Knoten, der aus Stahlsträngen zusammengewebt ist. Auf dem Schild daneben wird erwähnt, dass die Person, die es getan hat, es mit dem Wissen von Ingenieuren getan hat, die eindeutig wussten, dass Knoten unmöglich sind. Und doch steht es da, trotz der Überzeugung vieler Experten auf diesem Gebiet, es wurde getan. Die Wissenschaft ist nicht die Herrschaft des Mobs, der Sieg gehört denen, die nicht akzeptieren, was NICHT getan werden kann, sondern versuchen, herauszufinden, wie es geht.
@Arjang: Im Allgemeinen stimme ich Ihrem Standpunkt zu und es kam vor, dass die Mehrheit der Wissenschaftler in etwas falsch lag. Aber die großen Fortschritte in der Physik in der ersten Hälfte des 20. Jahrhunderts haben sie in einen Zustand gebracht, in dem Physiker alles verstehen können, was sie in die Finger bekommen können. Sie gehen mit LHC zu riesigen Energien, nur um einige neue Dinge zu finden, die sie studieren könnten. Es gibt einfach keinen Platz für eine neue physikalische Wechselwirkung, die unter normalen Bedingungen stattfindet, die von Tausenden von Physikern 100 Jahre lang irgendwie unbemerkt bleiben würde.
@gigacyan: Nach dem, was heute bekannt ist, ja, es ist unmöglich. Aber beim Ausprobieren würde man neue Dinge lernen und Fortschritte machen. Vielleicht nicht in der Kalten Fusion selbst, aber in anderen Bereichen. In der Mathematik war es am Ende nicht so wichtig, die schwierigen Probleme alleine zu lösen, aber auf dem Weg dorthin wurden viel mehr Werkzeuge / Entdeckungen gemacht. Schauen Sie sich zum Beispiel den letzten Satz von Fermat an, ob er wahr war oder nicht, war nicht so wichtig wie die Studienfächer, die von den Menschen erfunden wurden, die daran arbeiteten. Es geht nicht nur um das Endergebnis, sondern um den Weg dorthin.
@Arjang: Wenn Sie historische Beispiele mögen, nehmen Sie Perpetuum-Motion-Maschinen, Ihre Argumente können auch auf diese angewendet werden. Würden Sie sagen, dass wir die Möglichkeit des Baus einer solchen Maschine nicht ausschließen können, weil es möglicherweise physikalische Phänomene gibt, von denen wir nichts wissen und die es möglich machen würden?
Ich denke, irgendwann muss ein Schlussstrich gezogen werden. Aber die Frage ist, wer die Grenze ziehen sollte, wir, die sie nicht finanzieren, oder die Wissenschaftsgemeinschaft, die zum neuen Dogma geworden ist. Wenn man sich anschaut, wie viel Geld für Kriege verschwendet wurde, ist die gesamte Forschungsfinanzierung (sowohl wissenschaftlich als auch kurios) nichts. Wenn sich jemand entscheidet, Perpetuum Mobile weiterzuverfolgen, würde ich ihn auf das Bekannte und die Geschichte der Versuche hinweisen und ihm alles Gute wünschen. Ein Perpetuum Mobile, das Nullpunktenergie nutzt, eine neue Wendung? en.wikipedia.org/wiki/Zero-point_energy#Claims_in_pseudoscience
@Arjang, wenn sich irgendwann in der Zukunft durch wiederholte Experimente gezeigt hat, dass kalte Fusion und / oder ewig mobile Maschinen möglich sind, werden wir alle korrigiert. Bis dahin gibt es keinen Grund anzunehmen, dass beides möglich ist.
@Steven Devijver: Ist das alles, was sie tun? Dasselbe Experiment immer wieder wiederholen? Das ist Wahnsinn! Ich dachte, mit Recherche meinen sie zu variieren, zu modifizieren, neue Dinge zu verwirklichen. Natürlich ist es verrückt, dasselbe Experiment zu wiederholen und ein anderes Ergebnis zu erwarten, aber das sollte nicht Forschung genannt werden.
Es gibt einen großen Unterschied zwischen kalter Fusion und Multiversen: Kalte Fusion, falls vorhanden, sollte experimentell nachweisbar sein (tatsächlich behaupteten Pons und Fleischmann, genau das getan zu haben), würde aber gegen aktuelle Theorien verstoßen. Andererseits sind multiple Universen im Prinzip nicht beobachtbar, verletzen aber kein bekanntes Gesetz der Physik und können nur als falsch bewiesen werden, indem eine der physikalischen Theorien, die sie erklären sollen, als falsch bewiesen wird. Daher ist die Frage der kalten Fusion eine physikalische, während die Frage der Multiversen im Grunde eine philosophische ist.
@StevenDevijver: Perpetuum-Motion-Maschinen (die funktionieren) verstoßen per Definition gegen CoE. Was unter den Begriff "Kalte Fusion" fällt, ist in erster Linie eine Reihe von empirischen Behauptungen, die nach einer Erklärung suchen. Die nicht-trivialen Erklärungen, die einem leicht in den Sinn kommen, verletzen im Allgemeinen bekannte Gesetze der Physik. Aber die empirischen Behauptungen selbst sind nicht dieselben wie die Erklärungen, die wir finden. Kalte Fusions- und Perpetuum-Motion-Maschinen gehören damit in ganz unterschiedliche Kategorien.

Es gab kürzlich ein Perspektivpapier über die Natur:

https://www.nature.com/articles/s41586-019-1256-6

http://www.lenr.com.cn/uploadfile/2019/0531/20190531071312608.pdf

Sie behaupten, dass die Ergebnisse immer noch nicht vollständig reproduzierbar sind, aber die Experimente geben dennoch einige Einblicke in das Verhalten hochhydrierter Materialien.

Hier ist die Veröffentlichung, in der die Wurzeln der Studien von Rossi et al. liegen: Noninski VC, Fusion Technology, 21, 163–167 (1992). Anscheinend ist das wichtige Machtungleichgewicht, das in diesem Artikel gefunden wurde (veröffentlicht in einem seriösen Peer-Review-Journal), in einigen bisher unbekannten Aspekten der klassischen Physik zu suchen.

Sie wollen reich werden und versuchen, Geld zu verdienen. ;) Zwei Wege:

  1. Sie haben 'ein geheimes Gerät', das funktioniert -> Finanzierung, Patente, RICH,RICH,RICH ... (ich glaube nicht, dass sie dieses Geheimnis haben)
  2. Sie haben eine 'geheime Lüge' - Finanzierung, RICH (es war eine Risikooperation usw., Vertragsdetails, ... ruinierter Ruf ... aber RICH)

Ich werde kein direktes Urteil über falsch oder nicht falsch fällen. Ich werde abwarten und sehen. Vielleicht macht jemand eines Tages unerwartete Erfahrungen wie den jüngsten 'Anti-Laser', der Energie 'zerstört'. Um meine „operative“ Position zu rechtfertigen, kann ich eine Geschichte akademischer Arroganz erzählen: Die nationale Fernsehsendung begann hier 1955, 1957 wurde die russische Sputnic in den Orbit geschickt und praktischerweise erschien ein Akademiker im Fernsehen und sagte: „Das ist unmöglich, Russen sind Lügner.

* hinzugefügt : * gefunden:
Wie man Elemente mit Laserlicht umwandelt

Die kohärente photonukleare Isotopentransmutation (CPIT) erzeugt ausschließlich radioaktive Isotope (RIs) durch kohärente photonukleare (γ,n)- und (γ,2n)-Reaktionen über E1-Riesenresonanzen.

EDL - Proton-21 (Adamenko)

Das Hauptaugenmerk der EDL-Forschung liegt auf einem neu entwickelten und selbsterhaltenden Prozess, der durch eine kontrollierte Stimulation zum Kollabieren von kondensierter Materie führt. In diesem so erzeugten kollabierten Zustand wird die Wirkung der Coulomb-Barriere unbedeutend, und es findet eine schnelle Transmutation von Elementen und Isotopen statt, die beobachtet werden kann.

arxiv 2013/05 Update - Experiment 1 - Theorie 0
SIEBEN Forscher von Universitäten aus Italien und Schweden, nämlich Hanno Essén, berichteten kürzlich über einen Hinweis auf eine
anomale Wärmeenergieerzeugung in einem Reaktorgerät :

Eine experimentelle Untersuchung einer möglichen anomalen Wärmeentwicklung in einem speziellen Reaktortyp … In beiden Experimenten wurde eine anomale Wärmeentwicklung angezeigt. .. , das Ergebnis ist immer noch eine Größenordnung höher als bei konventionellen Energiequellen .

Anscheinend ist die Theorie in Schwierigkeiten.

Der arxiv-Link wurde über Wavewatching erhalten , dieselbe Website, auf der ich herausfand, dass der D-Wave-Quantencomputer gegen die Konsens-QM-Theorie arbeitet.